当前位置: 首页 > 专题范文 > 公文范文 >

英语高考北京模拟试卷及答案(范例推荐)

作者: | 发布时间:2023-05-14 16:45:17 | 浏览次数:

下面是小编为大家整理的英语高考北京模拟试卷及答案(范例推荐),供大家参考。

英语高考北京模拟试卷及答案(范例推荐)

英语高考北京模拟试卷及答案1

  第一部分:听力(共两节,满分20分)

  做题时,先将答案标在试卷上,录音内容结束后,你将有两分钟的时间将试卷上的答案转涂到答题卡上。

  第一节(共5小题;每小题1分,满分5分)

  听下面5段对话。每段对话后有一个小题,从题中所给的A、B、C三个选项中选出最佳选项,并标在试卷的相应位置。听完毎段对话后,你都有10秒钟的时间来回答有关小题和阅读下一小题。每段对话仅读一遍。

  1. What do we learn from the conversation?

  A. The man hates to lend his tools to other people.

  B. The man hasn’t finished working on the bookshelf.

  C. The man lost those tools.

  2. What do we know about the man?

  A. He doesn’t like his job.

  B. He will not give up his job.

  C. He has a large family to support

  3. What"s the relationship between the two speakers?

  A.Classmates. B. Teacher and student C. Headmaster and teacher.

  4. Who is worried about gaining weight?

  A.The son. B. Aunt Louise. C. The mother.

  5. Why doesn’t the woman buy the coat? .

  A.It is expensive. B. There isn"t her size. C. She doesn’t like the color.

  第二节(共15小题:每小题1分,满分15分)

  听下面5段对话或独白。每段对话或独白后有几个小题,从题中所给的A、B、C、D个选项中选出最佳选项,并标注在试卷的相应位置。每段对话或独白前,你将有时间阅读各个小题,每小题5 秒钟;听完后,各小题将给出5秒钟的作答时间,每段对话或独白读两遍。

  听第6段材料,冋答6、7题。

  6. What is the woman probably?

  A.A hotel clerk. B. A house agent. C. A shop assistant,

  7. What is the pillow filled with?

  A.Cotton. B. Dried flowers. C. A special material.

  听第7段材料,冋答8、9题。

  8. What kind of skills does the woman not have?

  A.Operating computers. B. Doing business. C. Typing.

  9.Which company did the woman work in?

  A. A trading company and a trust company.

  B. A trust company.

  C. A trading company.

  听第8段材料,回答10至12题。

  10. What"s wrong with the woman"s mother?

  A. She has been sick.

  B. She misses her family and friends.

  C.She can"t earn enough to support her family.

  11. Where does the woman live?

  A.In America. B. In India. C. In Britain.

  12. What does the woman plan to do next year?

  A. Study a new language.

  B. Travel to India.

  C. Visit her father"s native country.

  听第9段材料,回答13至16题。

  13. How many Economics lectures will the man attend every week?

  A. 5 times, from Monday to Friday.

  B. Two times, on Thursday and Friday.

  C. Two times, on Tuesday and Thursday.

  14. Why did the man miss the meeting for the new students yesterday?

  A. Because he hadn’t received any notice about that meeting.

  B. Because he had to attend the group discussion.

  C. Because he had to do some part time jobs yesterday.

  15. If a student wants to earn the scholarship, what is the required attendance rate?

  A. 80% B. 90% C. 100%

  16. Which of the following statements is not true according to the conversation?

  A. The man is a grade one student in the university.

  B. The man has to work after school.

  C. The man thinks the time of the lecture is too early.

  听第10段材料,回答17至20题。

  17. How long has the speaker lived in a big city?

  A. One year. B. Ten years. C. Eighteen years.

  18. What is the speaker"s opinion on public transport?

  A. It"s comfortable. B. It"s time-saving. C. It"s cheap.

  19. What is good about living in a small town?

  A. It’s safer. B. It’s healthier. C. It’s more convenient.

  20. What kind of life do the speakers seem to like most?

  A. Busy. B. Colorful. C. Quiet.

  第二部分:英语知识运用(共两节,满分35分)

  第一节单项填空(共15题;每小题1分,满分15分)

  请认真阅读下面各题,从题中所给的A、B、C、D四个选项中,选出S佳选项,并在答题卡该项涂黑。

  21. The death of the closest relatives, almost every one of us has experienced, always makes people feel bitter and unbearable.

  A. which. B. who C. as D. the one

  22. Purchases online are to be believed provided that you and the seller have communicated and reached a compromise the wishes of both parties. A. in defense of B. in view of C. in line with D. in company with

  23. there is sup* and demand, there is commerce.

  A. When B. Where C. Even if D. As if

  24.Despite his death, Castro"s courage and wisdom are always believed a new generation of political leaders in Latin America.

  A. to inspire B. inspiring C. to have inspired D. having inspired

  25. The fact that China was excluded from the negotiations of the Trans-Pacific Partnership (TPP) led many to believe that it was at least partially engineered to China’s economic rise.

  A. counter B. clarify C. conclude D. rebel

  26. 一I’ve heard that Miss Xue has been awarded first prize in the composition contest. What about you?

  —Oh, actually, it’s a piece of cake for her but for me.

  A. child’s play B. as easy as pie C. a cup of tea D. a hard nut to crack

  27. The Chinese embassy didn’t directly solve my problem, but I feel thankful that it me some advice, even if it wasn’t exactly conventional.

  A. had offered B. would offer C. offers D. offered

  28.In the deaf culture of North America, many listeners show not by clapping their hands but by waving them in the air.

  A. disapproval B. applause C. affection D. direction

  29. All rights reserved. Above the content does not have to reprint without myself-pertnission, thanks!

  A. arbitrarily B. randomly C. offensively D. optionally

  30. —Being young generations in China, they take up all of the challenges and opportunities that this country offers?

  一Absolutely.

  A. mustn’t B. mightn’t C. shouldn’t D. couldn’t

  31. Can Operation Mekong, a bloody and cruel and violent real-life tale,

  an entertaining 90 minutes in the movie theater?

  A. account for B. push for C. make for D. head for

  32. With millions of graduates and unemployed recent graduates hitting the job market, 2016 is to be yet another year of exceedingly difficult job hunting for young Chinese.

  A.bound B. essential C. fundamental D. sensitive

  33. He lived outside conventional ideas, he implied, because “I live alone mostly, in the middle of .”

  somewhere B. nowhere C. anywhere D. everywhere

  34. He’s never thought of getting addicted to drugs and destructive clangers they will bring him.

  A.how B. whether C. what D. whose

  35. To realize his dream as a pianist, he practicing playing it in all his free time.

  A.hired himself out B. threw himself into

  C.resigned himself to D. wrestled himself with

  第二节:完形填空(共20小题:每小题1分,满分20分)

  请认真阅读下面短文,从短文后各题所给的A、B、C、D四个选项中,选出最佳选项,并在答题卡该项涂黑。

  Hiro walked up to the microphone. He looked out at the huge crowd and felt a heaviness in the pit of his stomach. They were all 36 at him, waiting.

  “Uh, hi,” Hiro said into the microphone. For a moment, his mind went 37 , Then he saw Tadashi’s face in the audience. He was smiling and giving Hiro a thumbs-up.

  Hiro smiled back and took a 38 . “Sorry. My name is Hiro Hamada, and I’ve been working on something I think is pretty cool. I hope you like it.”

  Hiro put on a headset and reached into his hoodie(连帽衫). He 39 a small object no bigger than a paper clip. “This is a microbot.” The small object in his hand took a bow. “It doesn’t look like much but when it links up with the rest of its fellows, things get a little more interesting.”

  The crowd didn’t seem 40 . Then a murmur rose as the audience noticed waves of tiny mircrobots twisting across the floor. The single microbot flew from Hiro’s 41 and joined a towering column of microbots that had now formed onstage.

  Hiro smiled and tapped his headset. “The microbots are 42 with this neural transmitter(神经传导器).” He took the headset off and the microbots collapsed to the floor. They reformed into a column as soon as Hiro put the headset back on.

  “I think of what I want them to do,” Hiro said, “and they do it!” The microbots took the 43 of a hand waving. Everyone in the audience smiled and waved back. “The 44 of this tech are limitless. Take construction.”

  Hiro stared at the microbots, and with a wave of his hand, they picked up cinder(煤渣)blocks and assembled them into a tower. “ 45 used to take teams of people working by hand for months or years can now be 46 by one person!”

  “And that’s just the 47 Hiro said as he jumped off the tower. Everyone gasped, thinking he was about to 48 ,but the microbots rose and 49 him midair. Hiro smiled. He could see Tadashi giving him another 5O .

  “How about 51 ?”Hiro asked. The microbots transformed into a set of legs that 52 Hiro through the audience. “Microbots can move anything, anywhere, 53 . ”

  As they approached the stage, the microbots formed a set of 54 so Hiro could climb back up to it. “If you can think it, the microbots can do it!” Hiro said.

  The audience was with him now. The moment belonged to Hiro, and he was on a roll. “The only limit is your 55 ! Microbots!” he exclaimed, and the audience burst into applause.

  36.A. smiling B. shouting C. staring D. glancing

  37.A. open B. blank C. down D. away

  38.A. sigh B. look C. seat D. breath

  39.A. put away B. took out C. picked out D. set in

  40.A. worried B. exhausted C. delighted D. impressed

  41.A. hand B. pocket C. position D. headset

  42.A. charged B. controlled C. constructed D. conducted

  43.A. picture B. place C.shape D. sight

  44.A. applications B. instructions C. meUiods D. meanings

  45.A. It B. There C. What D. That

  46.A. destroyed B. accomplished C. accessed D. occupied

  47.A. story B. feeling C. challenge D. beginning

  48.A. fall B. break C. leave D. decline

  49.A. sought B. caught C. chased D. stopped

  50.A. thumbs-up B. look C. applause D. reward

  51.A. operation B. construction C. transportation D. reception

  52.A. removed B. walked C. ran D. led

  53.A. with caution B. without hesitation C. without order D. with ease

  54.A. legs B. blocks C. stairs D. stages

  55.A. imagination B. determination C. courage D. confidence

  第三部分:阅读理解(共15小题;每小题2分,满分30分)

  请认真阅读下列短文,从短文后各题所给的A、B、C、D四个选项中,选出最佳选项,并在答题卡该项涂黑。

  A

  New Scientist Event

  Instant Expert— How Your Brain Works

  Saturday, 20 May 2017from 10:00 to 17:00

  London, United Kingdom

  Ever wondered how your brain works? How that stuff in your head enables you to see, hear and think about the world around you, make decisions and act on them? Join six leading scientists to explore what we know about 4fcthe most difficult kilo of matter in the universe”.

  The day will be chaired by psychologist and presenter of BBC Radio 4’s All in the mirtd,Claudia Hammond. ’

  Overview:

  The brain has long been a source of fascination. In 1819, the radical(激进的)thinker and surgeon William Lawrence put it like this: “It’s strongly suspected that a Newton or Shakespeare excels other men only ... by having an extra inch of brain in the right place.”

  Today, many such suspicions are certainties. This event will introduce you to the evolution of the brain. We’ll study technique for controlling the brain using electric and magnetic fields, as well as the latest technologies that follow you to control the outside world using your mind alone.

  l Topics covered will include:

  l Evolution of the brain

  l How memory makes us human

  l Emotion

  l Sleep and brain

  Booking information:

  l The event will be held in Brunei Auditorium at RCGP/30 Euston Square. And doors to the Auditorium will open at 9:15 am.

  l We require the name of each person attending — please ensure this is provided at the time of booking.

  l The schedule for the day will be confirmed closer to the event, and will be emailed to all ticket holders.

  l A minimum of 100 early bird discounted tickets are available priced at £129 (saving £20 on the full ticket price of £149).

  56. What can attendees learn from the event?

  A. How much their brain weighs.

  B. How their sleep is related to their brain

  C. Whether they can act on decisions they made.

  D. How their brain is controlled by electric and magnetic fields.

  57. When people book tickets, they are .

  A. required to offer their email address

  B. likely to be told the schedule for the event

  C. required to decide when to enter the Auditorium

  D. likely to enjoy a discount if they promise to come to the event early

  B

  How many really suffer as a result of labor market problems? This is one of the most critical yet debatable social policy questions.

  In many ways, our social statistics overstate the degree of hardship. Unemployment does not have the same horrible consequences today as it did in the 1930’s when most of the unemployed were primary breadwinners, when income and earnings were usually much closer to the margin of survival, and when there were fewer effective social programs for those failing in the tabor market. Increasing wealth, the rise of families with more than one wage earner, the growing dominance of secondary earners among the unemployed and improved social welfare protection have unquestionably relieved the consequences of joblessness. Earnings and income data also overestimate the scale of hardship. Among the millions with hourly earnings at or below the minimum wage level, the majority are from multiple-eamer, relatively well-off families. Most of those counted by the poverty statistics are elderly or handicapped or have family responsibilities which keep them out of the labor force, so the poverty statistics are by no means an accurate indicator of labor market"problems.

  Yet there are also many ways our social statistics underestimate the degree of labor-market-related hardship. The unemployment counts exclude the millions of fully employed workers whose wages are so low that their families remain in poverty. Low wages and repeated or long-time unemployment frequently interact to weaken the capacity for self-support. Since the number experiencing joblessness at some time during the y^ar is several times that unemployed in any month, those who suffer as a result of forced idleness can equal or exceed average annual unemployment, even though only a minority of the jobless in any month really suffer. For every person counted in the monthly unemployment totals, there is another working part-time because of the inability to find full-time work, or else outside the labor force but wanting a job. Finally, income transfers in our country have always focused on the elderly,disabled, and dependent, neglecting the needs of the working poor, so that the dramatic expansion of cash and non-cash transfers does not necessarily mean that those failing in the labor market are adequately protected.

  As a result of such conflicting evidence, it is uncertain whether those suffering seriously as a result of labor market problems number in the hundreds of thousands or the tens of millions, and, hence, whether high levels of joblessness can be tolerated or must be counteracted(抵消)by job creation and economic stimulation. There is only one area of agreement in this debate~that the existing poverty, employment, and earnings statistics are inadequate for one of their primary applications, measuring the consequences of labor market problems. ’

  58. In Pararaph 2, the author contrasts the 1930’s with the present in order to show that .

  A. more people were unemployed in the 1930,s

  B. unemployment is more intolerable today

  C. social programs are more in need now

  D. income level has increased since the 1930’s

  59. Which of the following is true according to the passage?

  A. A majority of low-wage workers receive earnings from more than one job.

  B. Repetition of short-term unemployment mainly contributes to people’s loss of working capacity.

  C. Many unemployed people are from families where other members are working.

  D. Labor market hardship is understated because fewer individuals are jobless than counted.

  60. It can be inferred from the passage that the effect of income transfers is often not felt by .

  A. those doing a low-paid, part-time job

  B. children in single-eamer families

  C. workers who have just retired

  D. full-time workers who become unemployed

  61. Which of the following is the principal topic of the passage?

  A. What causes labor market problems that result in suffering.

  B. Why income statistics are imprecise in measuring degrees of poverty.

  C. When poverty, employment, and earnings figures agree with each other.

  D. How statistics give an unclear picture of the labor-market-related suffering.

  C

  Fed up with constantly having to recharge or replace batteries in your ever-expanding electronic devices? The solution may be just a few steps away.

  “Energy harvesting” promises to power countless consumer devices, often with nothing more than your body’s movement or heat. Dozens of companies around the world already offer such products, but many experts believe the market for the technology could explode due to electronic devices being developed for the Internet of Things.

  “It’s huge,” said Graham Martin, CEO of the EnOcean Alliance, a San Ramon-based group of businesses that promotes wireless energy-harvesting technologies. With the Internet of Things expected to combine billions of devices, “if they are all battery-powered, we’ll have a problem because there’s not enough lithium(锂)in the world,” he added. “So a lot of them will have to use energy harvesting.”

  Among the most basic forms of the technology is body power. When certain materials are squeezed or stretched, the movement of their atoms creates an electrical charge. Automatic watches have employed the concept for decades, for example, by winding themselves when their user moves their arm. Now, the concept is being considered for a number of other devices.

  In a contest seeking visionary ideas for wearable technologies, Intel awarded $5,000 for a concept to change the temperature difference between a person’s body and a special piece of clothing they’d wear into electricity for mobile devices.

  Using sound to power devices is another energy-harvesting variation. Stanford University engineers are testing smart microchips that create electricity from ultrasound to power implantable devices that can *yze a person’s nervous system or treat their diseases.

  A textile research association in Spain is proposing to obtain electricity from radio waves that flow around everyone to power sensors sewn into clothes, which can monitor a person"s heartbeat or other vital signs. Research firm lDTechEx has estimated that annual global sales of energy-harvesting products could hit $2.6 billion by 2024, while WinterGreen Research predicts sales of $4.2 billion by 2019.

  Obtaining stable energy from devices can be complex, however. For one thing, the motion that generates the electricity has to be constant to be useful. Moreover, the amount of power the devices produce depends on the person using them,according to a Columbia University study. It determined that taller people on average provide about 20 percent more power than shorter ones when walking, running or cycling.

  It’s also unclear how eagerly consumers might welcome energy-harvesting products. While such devices are expected to cost less than battery-powered alternatives when compared over many years, experts say, people may continue buying ones with batteries merely because those would be cheaper in the short term.

  62. Which “explode” in the following sentences has the most similar meaning to the word “explode” in Paragraph 2?

  A. They were clearing up when the second bomb exploded.

  B. The continued tension could explode into more violence.

  C. The population exploded to 40,000 during the last tourist season.

  D. The boss exploded when he heard of the resignation of the secretary.

  63. What makes “energy harvesting” necessary according to the passage?

  A. The waste of lithium in the world.

  B. The increasing number of electronic devices.

  C. The development of technology.

  D. The pollution caused by batteries.

  64. It can be learned from the passage that .

  A. energy-harvesting products save money in the long run

  B. taller people can surely produce a larger amount of power

  C.automatic watches harvest energy from the users’ body heat

  D.two ways of harvesting energy are mentioned in the passage

  65. Which of the following might be the best title for the passage?

  A. Energy harvesting: a low-risk technology

  B. Energy harvesting: a high-profit technology

  C. Energy harvesting: a problem-free technology

  D. Energy harvesting: an environment-friendly technology

  D

  He fascinated Victorian England with his unequalled skill at brilliant cases, based on logical reasoning and grasp of forensic(法医的)science, not to mention a mastery of disguises(伪装)and an encyclopedic knowledge of the criminal underclass.

  But this detective was not Sherlock Holmes but a real life investigator, Jerome Caminada, who, new research suggests, helped inspire Sir Arthur Conan Doyle’s celebrated hero.

  A biography of Caminada written by Angela Buckley reveals a series of striking similarities between him and the fictional character, in terms of their unusual methods and character. It also establishes strong echoes between the real detective’s cases and plot lines used by Doyle.

  The son of an Italian father and Irish mother, Caminada was based in Manchester, but was involved in cases which took him across the country, and he enjoyed a nationwide profile in the press, where accounts of his legend were widely reported.

  Most of his career was spent with Manchester City Police Force although he later operated, like Holmes, as a “consulting detective”.

  He became well-known in the mid 1880s, shortly before Sherlock Holmes made his first appearance in A Study in Scarlet and parallels soon emerged between the two.

  As the fictional character relied on a network of underworld contacts~the Baker Street Irregulars~so Caminada was known for his extensive web of informers, whom he would often meet in the back seat of a church. •

  These characters helped him build up an encyclopedic knowledge of the Criminal fratemity(兄弟会), among whom he would often move in disguise—another method in coptnQn with Holmes.

  His skill with disguises was so renowned that on one occasion, while tracking a group of thieves at the Grand National dressed as a laborer, his own chief police officer was unable to recognise him.

  . However, he also posed as white collar professionals, once while bringing a bogus(假的")doctor to justice.

  Over the course of his career, he was reportedly responsible for the imprisonment of 1,225 criminals. His most famous case—and perhaps the one which most closely resembles a Holmes story—was “Mystery of the Four-Wheeled Cab”.

  Mrs Buckley identifies Caminada’s “Moriaty” figure as Bob Horridge, a violent, intelligent career criminal, with whom he had a 20-year fight, which began when Caminada arrested him for stealing a watch, landing him with a sentence of seven years’ penal servitude because of his previous crimes.

  This harsh sentence for a relatively small crime angered Horridge so much that, as he was sent down, he swore revenge(报复)on the detective.

  On his release, Horridge’s criminal enterprises grew in size and scope, but he was usually able to stay one step ahead of the authorities, often effecting dramatic escapes.

  His good luck finally ended after he shot two police officers. Caminada tracked him to Liverpool where the detective, disguised once more, eventually arrested him. Horridge was convicted of attempted murder and sentenced to life imprisonment.

  Caminada’s “Irene Adler” was Alicia Ormonde, an apparently well-educated woman with a noble background and expensive tastes, who was actually an experienced criminal wanted across the country for A^uds and thefts.

  Caminada tracked her down and arrested her, but—in an echo of Holmes’ fascination with Adler~the detective apparently became attracted by her. The case took place in 1890, a year before Adler appeared in A Scandal in Bohemia.

  Caminada—who published his memoirs on retiring~died in 1914, the year the last Holmes book was set.

  Other individuals have previously been put forward as the basis for Holmes. However, Mrs Buckley, whose book is called The Real Sherlock Holmes, believes that Caminada was used to give Holmes a better grounding in actual casework among the criminal fraternity, inspiring his detecting styles and some of the puzzling cases he encountered.

  66. From the passage we can know that Caminada .

  A. died after the last Holmes book was set

  B. was the only basis for Holmes

  C. had a good knowledge of forensic science

  D. had a whole life career as a “consulting detective”

  67. Which of the following CANNOT prove that Caminada was the basis for Holmes?

  A. Caminada became a national figure shortly before Holmes’ first appearance.

  B. Caminada tracked an attractive and talented criminal, similar to Irene Adler.

  C. Caminada was known for his extensive web of informers in the Baker Street.

  D. Caminada had a Moriaty-like enemy for a long period of time.

  68. Which of the following sentences could be added into the blank part?

  A. His most famous case was the Manchester Cab Murder of1889.

  B. As an investigator Caminada served with the police between 1868 and 1899.

  C. Following his retirement, he published a second volume under his own name.

  D. Other disguises included as drunken down and outs, as well as working class roles.

  69.Caminada used to disguise himself in the following cases EXCEPT .

  A. bringing a bogus doctor to justice

  B. tracking and arresting Alicia Ormonde

  C. tracking a group of thieves at the Grand National

  D. arresting Horridge and sending him to life imprisonment ^

  70. The most suitable title of the passage should be .

  A. Detecting styles of Sherlock Holmes B. Caminada and hrs legend

  C.Detectives and their disguises D. Has the real Holmes beert deduced(演泽)?

  第四部分:任务型阅读(共10小题;每小题1分,满分10分)

  请认真阅读下列短文,并根据所读内容在文章后表格中的空格里填入一个最恰当的单词。

  注意:请将答案写在答题卡上相应題号的横线上,每个空格只填一个单词。

  How traffic affects our health

  When talking about traffic problems, we,Jso need to take into consideration the pollution that we are exposed to and the health problems traffic causes. Given below is an in-depth exposition on how traffic afftcts our health and what we can do to avoid some of these effects to live a longer and healthier life.

  Aches and Pains

  Driving in a slouched(无精打釆的)position puts pressure on your back and neck. Back pain is often due to poor posture which can have a neu,nti\ e effect on your health. Your bad posture also puts stress on the joints of your elbows and knees, causing pain and tiredness to these areas of your body.

  When you are driving in traffic, you are constantly shifting between the accelerator and brake (in automatic transmission cars), and clutch, brake, and accelerator (in manual transmission cars). This constant shifting puts pressure and stress on your knees, causing knee-related problems and aches.

  What can you do?

  You can counteract these ill effect and posture problems by sitting upright and having a support for your lower back and neck. Adjust the position of your seat in such a way that you can drive at ease without any discomfort. The backrest of the seat should be straight up, and not pulled down too much. Respiratory(呼吸的)Problems

  Exposure to heavy traffic implies exposure to poisonous rases that are emitted by vehicles. It is believed that the transportation sector in the US is resporbible for a quarter of the country’s greenhouse gas emissions.

  Our lungs arc like sponges, soaking in everything that is breathed in by our respiratory system. The smoke coming out of the tailpipes of vehicles presents a serious threat to human health. Prolonged exposure to traffic emissions can cause,vious respiratory diseases including lung cancer.

  What can you do?

  There are ways to avoid breathing in polluted air If you can, avoid driving in parts of your town with heavy traffic congestion. However, if you find yourself driving through such areas, roll up your car windows to minimize exposure to harmflil gases. If you plan to go out for a walk or run, do it in a place where you get loads of fresh air. Eat foods that are rich in antioxidants(抗氧化剂),and exercise regularly so that your immune system is strong enough to fight off potential threats to your health.

  Travel and traffic are two things that are absolutely unavoidable,and that is why we should try to be as comfortable as possible, to be stress-free and illness-free!

  How traffic affects our health

  Passage outlineSupporting details

  Lead-inWhen it (71) to traffic problems, we should pay enouch attention to the pollution we are exposed to and the health problems that traffic causes.

  Aches and PainsProblems(72) a Door Dosition when you are driving can affect your health badly in several ways such as back, neck, even the joints of your elbows and knees.

  Shifting posture constantly in the (73) of driving will put pressure and stress on your knees, causing aches and pains.

  SolutionsYou can sit upright and have a support for your lower back and neck, j You can make (74) to the position of your seat.

  You can (75) the backrest of vour seat and avoid pulling it down too much.

  Respiratory

  ProblemsProblemsIt is believed that the transportation sector is to (76) for a quarter ~greenhouse gas emissions in the US.

  Our lungs breathe in harmful elements in the smoke from vehicles, damaging our health seriously.

  Long exposure to dangerous pollutants from vehicles can cause various lung-related diseases, lung cancer (77) .

  SolutionsYou should try to drive through the areas (78) from heavy traffic congestion.

  You must wind up your car windows to keep exposure, to harmfijl pollutants to the (79) .

  You should choose a place where you get loads of fresh air to take a walk or have a run.

  You must attach (80) to your diet and regular exercise to fight off potential threats to your health.

  Conclusion t We should try to be stress-free and illness-free while driving or I traveling.

  第五部分:书面表达(满分25分)

  阅读下面的短文,然后按要求写一篇150字左右的英语短文。

  In campus violence cases in Beijing during the last five years, 14 percent of the offenders not only beat others, but also insulted(侮辱)them. They slapped(打耳光)others, made them kneel down, and in some cases took off other people’s clothes.

  Violence at schools has been widely reported in media in recent years 一 the violence among teenagers has drawn public and government attention. On Nov 11, the Ministry of Education, along with eight other central sectors, published a guideline on dealing with school violence.

  The guideline stressed that students with serious behavioral problems should be handed over to special schools. Or, in more serious cases, they may take criminal responsibility and be sent to prison.

  “The common tolerant attitude toward violence should be changed,” wrote Jiaxing Daily. “When teenagers do something wrong, we should tolerate and help them when necessary. But all of this has a limit."

  [写作内容]

  1.以约30个词概括上文的内容要点:

  2.然后以约120个词写作以下相关内容:

  (1)校园暴力存在且越来越严重的原因(至少两点);

  (2)你认为如何应对校园暴力(至少两点);

  [写作要求]

  1.不得直接引用原文中的句子;

  2.文中不能出现真实姓名和学校名称;

  [评分标准]

  概括准确,语言规范,内容合适,篇章连贯。


英语高考北京模拟试卷及答案扩展阅读


英语高考北京模拟试卷及答案(扩展1)

——届北京市高考语文模拟试卷题目及答案

届北京市高考语文模拟试卷题目及答案1

  第I卷 阅读题(70分)

  一、现代文阅读(35分)

  (一)论述类文本阅读(9分)

  阅读下面的文字,完成1—3题。

  文字是记录和传达语言的书写符号。语言是有声的思维,文字是有形的思维。二者都是思维的外化。像语言一样,文字也是思维能力的反映,蕴含着思维的特点。从某种意义上说,古文字是对思维进行考古的工具。

  古人类主要运用形象进行思维。这种特点反映在古诗歌中,人们喜欢用比兴来说明某种道理;反映在原始神话中,人们通过具体的故事来说明某个道理,抽象的道理是没用的。形象思维曾是人类最得心应手的思维方式。抽象思维是在形象思维的基础上逐步发展起来的。从文字的创造特点看,绘画文字是最早的文字。这种文字的主要特点是象形。不仅中国的古文字用形象来表示,古巴比伦人、古雅玛人、古埃及人等无一不是从象形开始来创造古文字的。 创造文字的时代,人们抽象思维的能力还比较薄弱。人们进行抽象思维,还很难脱离具体形象来进行,往往要依赖于对形象的类比、比较、联想来进行。表现在文字的创造上,形与义密切结合在一起,人们是从形象来了解符号的意义的。文字的产生一方面是以人类抽象思维的能力达到一定水*为基础的。但另一方面,象形文字又标志着人类抽象思维能力还不发达,这种低下的抽象思维能力不可能创造太抽象的符号。

  抽象思维能力的低下还主要表现在古文字上的词类上,表现为形容词的极端贫乏。从甲古文看,形容词数量相当少,只有白、厚、明、光、赤、美、丽等为数不多的形容词。事物的性质包含在事物之中,与事物不可分离,只能通过人的思维才能把事物的性质从事物本身分开。在近代一些少数民族那里,要说热,只能说“像火一样”。

  早期文字中的“鱼”字是各种各样的鱼类象形,后来逐渐概括出一种共同的鱼的特征的象形文字,作为各种各样鱼的代表。早期文字中,几乎每个文字都经历了一定的概括过程。没有概括过程,就不可能产生一般意义上的文字。这种概括能力使人们能概括出事物的共同性质,撇开了同类事物之间的差别性、个性和偶然性,产生了最初的种、类概念。

  在概括能力发展的同时,形象思维本身也在不断发展着。形象思维从自身中演化出了抽象思维。抽象思维的发展又推动着形象思维的发展。古人类从事物中抽象出事物的性质,从而创造了形容词后,又创造着代表新的形象的词。例如,从羊的肥大的形象创造了“美”字,进一步又创造了“美服”“美女”等新的名词。形容词产生后,也使原有的一些代表具体形象的词成为更形象生动的新词。例如月字、镜字,在抽象出了“明”字后变成了“明镜”“明月”,使形象更逼真。古人类从众多的个别的事物中概括出代表一般意义的词后,又以此为指导去认识新的个别事物,创造新的字。

  从古文字的演变进化过程中,我们可以看到形象思维与抽象思维在互相补充、互相促进中辩证地发展,使人类的思维能力不断由低级向高级发展,从而使人类能够更探更广地认识世界。 (摘编自朱长超《从古文字看原始思维及其发展》,有删改)

  1.下列关于原文内容的表述,不正确的一项是(3分)

  A.“语言”与“文字”都是思维的外化,如同可以通过古代语言来考察原始思维一样,我们也可以从古文字来考察原始思维。

  B.创造文字的时代里,人们常需要依赖形象的类比、比较、联想来进行抽象思维。那时候,

  人们从形象来了解符号的意义。

  C.概括让人们认识到了事物的共性,没有概括就不可能产生一般意义上的文字,因而几乎每个文字都经历了一定的概括过程。

  D.古人类充满智慧,能够从众多个别的事物中概括出代表一般意义的词之后,又以此为指导去认识新的个别事物,创造新字。

  2.下列理解和分析,不符合原文意思的一项是(3分)

  A.因为文字是有形的思维,是思维能力的反映,蕴含着思维的特点,所以从某个意义上而言,古文字是对思维进行考古的工具。

  B.人们在古诗歌中喜欢用比兴来说明某种道理,在原始神话中用具体故事来说明某种道理。这种形象思维的主要特点便是象形。

  C.事物的性质与事物不可分离。形容词是对事物性质的抽象概括,当人们具备一定的抽象思维能力之后,形容词才有可能出现。

  D.形象思维与抽象思维关系密切、辩证发展。形象思维从自身中演化出了抽象思维,抽象思维的发展又推动着形象思维的发展。

  3.根据原文内容,下列理解和分析不正确的一项是(3分)

  A.古文字是考察原始思维的宝贵资源,因而,当下对古文字的研究与保护,不仅是对人类文化的尊重,更是对人类历史的尊重。

  B.古中国人、古巴比伦人、古雅玛人、古埃及人都是从象形开始创造古文字。这说明古代东西方人的思维中存有相同的因子。

  C.文字的产生是以人类抽象思维能力达到一定水*为基础的,因而,如果人类抽象思维的能力不太发达,人类就不太可能创造出文字。

  D.抽象思维与形象思维在相互补充、相互促进中辩证地发展,推动人类思维能力不断发展,使人类更好地认识世界。

  (二)实用类文本阅读(12分)

  阅读下面的文字,完成4—6题。

  梁思礼与梁启超

  解璋玺

  梁思礼生于1924年8月24日。这一年,或可称为梁启超的“灾年”。他的第一任夫人李蕙仙因病离世,而丧事初了,子又远行留学。因而老年得子,算是给他一点难得的慰藉,于伤痛之中还能体会到人生未尽之乐。

  梁启超固深爱着他的九个儿女,但爱小儿子梁思礼尤甚。这种人之常情是不难为人所理解的。他称小儿子“老baby(老白鼻),在写给儿女的信中常常提到这个聪明可爱、讨人喜欢的小儿子,以至于姐姐、哥哥们责怪“爹爹信中只说老白鼻不说别的弟妹,太偏心”,结果,梁启超特意在信中写了一大段文字,专讲达达、六六、司马懿(七女)的近况。不过,梁启超并不回避他是“专一喜欢老白鼻”的。他注意到老白鼻一点一滴的进步和变化,不到两岁,就已经“一天到黑„手不释卷‟”了,而且能唱“葡萄美酒”。梁启超在饮冰室读书、写作时,是不允许孩子们来打扰的,但老白鼻是个例外,每天“总来搅局几次”,梁启超不仅不恼,甚至“盼望老白鼻快来”,因为老白鼻的到来,恰恰是他“最好的休息机会”。过生日了,大孩子都不在身边,他就叫老白鼻代表姊姊、哥哥们拜寿。老白鼻也乐得如此,居然“一连磕了几十个响头,声明这是替亲家的,替二哥、三哥乃至六姊的”,而作为父亲,他在信中告诉大孩子们,“我都生受你们了”。这种父子之间其乐融融的场面,不仅儿子是天真烂漫的,父亲又何尝不是和蔼可亲,一团孩子气呢。

  梁启超病逝时梁思礼只有四岁半。母亲与姊姊、哥哥们合力供他读完了小学、中学,太*洋战争爆发前夕,母亲筹措了四百美金,送他到美国求学。其实,梁启超早在1925年为几个孩子预筹学费时曾说过:“至于老白鼻那份,我打算不管了。到他出洋留学的时候,有姊姊哥哥,还怕供给他不起吗?”然而,中日战争爆发后,他的姊姊、哥哥们陆续流落大后方,自顾不暇,哪里还有余力顾及远在天津的这个小弟弟?

  梁思礼到美国后,考入普渡大学电机工程系。由于家道中落,要靠家庭资助完成学业已不可能,他只能靠自己打工赚钱,交学费和维持生活。 “他凭着自己的聪明才智和苦斗精神,在国外打工读书,在饭馆里洗碗碟,在游泳池当救生员,他什么都能干”,苦读了八年。在1945年从普渡大学毕业,获得学士学位,随后,他又获得辛辛那提大学硕士学位和博士学位。梁家子弟没有不是心怀报国热忱的,虽然著名的无线电公司RAC向他发出了邀请,并许诺优厚的待遇,但父亲的话言犹在耳:“人必真有爱国心,然后方可用大事。”于是,他毅然谢绝了美国公司的邀请,和姊姊梁思懿同船回到祖国,投身于新中国的建设。

  几十年来,呕心沥血,筚路蓝缕,屡败屡战,排除万难,终于将远程运载火箭研制成功。中国能够成为世界上的航天大国,梁思礼作为开拓者中的一人,功不可没。

  1987年,他被选为国际宇航科学院院士,并担任国际宇航联合会副*;1993年,又当选为中国科学院院士,成为梁氏一门三院士①中的一位。

  (选自《新华文摘》,有删改)

  【注】①一门三院士,九子皆才俊。梁启超有九个子女,除梁思礼外,还有两位院士:梁思成,建筑学家;梁思永,考古学家,均为中央研究院院士。此外,梁思顺,诗词研究专家;梁思忠,西点军校毕业,参与淞沪抗战;梁思庄,图书馆学家;梁思达,经济学家;梁思懿,社会活动家;梁思宁,曾参加新四军。

  4.下列对材料有关内容的概括和分析,最恰当的一项是(3分)

  A.1924年是梁启超的“灾年”:妻子因病去世,丧事初了,爱子远行留学。梁思礼的出生,消解了他所有的抑郁情绪。

  B.梁思礼也许没能直接受教于梁启超,但梁启超给他的童年创造了一个健康、快乐的成长环境。他从父亲的言传身教中自会受到春风化般的滋润和熏陶。

  C.梁启超在写给儿女的信中常常提到讨人喜欢的小儿子,以至于哥哥、姊姊们责怪父亲不关心出国的子女,但是,梁启超没有作回应。 [来源:学,科,网Z,X,X,K]

  D.梁思礼历时几十年将远程运载火箭研制成功,使中国成为了世界上的航天大国。同时,因贡献突出,被评为中国科学院院士,成为梁氏一门三院士中的一位。

  5.结合全文,请简要分析梁思礼具有哪些精神品质?(4分)

  6.每当我们追念起梁启超继而谈到“一门三院士,九子皆才俊”时,心中总是为梁启超对儿女的成功教育而叹服。请结合文本谈谈你的看法。(5分)

  (三)文学类文本阅读(14分)

  阅读下面的文字,完成7—9题。

  借给你的温柔

  江文胜

  这天晚上,天下大雪,出奇的冷。养猪专业户王老汉蜷缩在家里的火盆前,他不敢出门。 忽然,狗叫。王老汉极不情愿地打开屋门,一股刺骨的寒风迎面刮来,他打了一个激灵。王老汉哆哆嗦嗦地来到院子里,没见到任何人,可狗还在叫。

  院子左边的一排房屋,是猪圈。他逐一查看猪圈,仔细地数,总共60头猪连毛发都没少一根。猪没被盗,狗为啥叫呢?肯定有情况,不然,通人性的狗是不会无缘无故地瞎叫的。果

  然,王老汉在那排距自己的住房最远的饲料库发现了“敌情”:喂猪的苕片饲料少了一袋! 王老汉立马来了精神,他赶紧回到屋里,拉上三十岁出头的儿子金华出来寻找。原来下了雪,现在雪停了,院子里从院门到饲料库、从饲料库到院门各有两行脚印,清晰可辨。金华说:“咱们跟着脚印去追那偷饲料的人!”王老汉说:“这是一个人的脚印,咱俩去追,可以对付,铁证如山,看他如何狡辩。”

  父子俩一前一后顺着脚印一口气追到了蜡子山矿区宿舍。脚印通到一间砖房的门前没了,别的砖房都熄了灯,只有这间房没有熄灯。

  真是胆大包天啊!做了贼还敢亮着灯?王老汉这样想。

  窗是玻璃窗,玻璃上结满了冰花。左下角的一块玻璃没了,糊上了报纸。王老汉用舌头舔湿了窗户纸,用手指轻轻一捅,报纸上便出现了一个小孔:一个黑黑的瘦瘦的中年汉子,坐在屋里的矮凳上,他的头上还冒着热气。啊!那袋装有红苕片的大编织袋赫然放在他的面前。床沿坐着一个双手笼在袖头里的老头,床头的被子下露出一个小脑袋,一个中年妇女站在这个黑汉子的对面。

  王老汉父子猫着腰蹲在窗户外,大气不敢出,倾听着屋里的动静。

  “粮,借来了。这就是,红苕片,100斤。你还愣着干啥?还不快去熬苕片汤!”汉子的声音。

  “妈,快熬,我饿坏了,我要吃苕片汤。”小孩的声音。

  “亚龙,这粮是咋借来的?”老人的声音。

  “爹,我事先写好了借条,借条压在袋子下。”汉子的声音。

  王老汉一愣,心想,我怎么没注意到那借条!真的有?

  “这不是偷吗?自打你从娘胎里出来,我是怎么指教你的?做人要本分!还好,留下了借条,人要凭良心。但借粮容易还粮难哪!矿上不开工资,啥时候能还上?”老人的声音。 “爹,这个我想过。车到山前必有路,活人不能让尿憋死,我还有一双手。到春天,矿上如果还不能开支,我就到农村去帮工。听说王老汉厚道,日子也过得殷实,不然*吗要借他的。我想去他那里,帮他干活,用工钱抵粮钱……”汉子的声音。

  “快走!”王老汉在心里喊。他拉起儿子往家里赶。进了院子,王老汉急奔饲料库,果然发现了那张借条:

  家里无米下锅。借一袋红苕干片。春天还。蜡子山矿区吴亚龙,此据。

  某某年某月某日

  上面还有一个用墨水作印泥盖的“吴亚龙”的私章。

  太阳从蜡子山上露出头来。吴亚龙推开屋门,见门口堆了一些东西:一袋大米、一袋面粉、一大块猪肉、几条鲜鱼和一袋冻豆腐……吴亚龙像见到了天外来物似的翻来覆去地找,竟找不出一张哪怕是手指宽的字条……

  7.下列对这篇小说思想内容和艺术特色的分析,最准确的一项是(3分)

  A.小说以“借给你的温暖”为题,揭示了文章的主题,表现了淳朴的人性美。

  B.小说运用了外貌、神态、动作、心理、对话、细节等描写手法,生动形象地刻画了王老汉和吴亚龙的形象。

  C.这篇小说讲了一个感恩的故事,他把人物放在大饥饿的时代,用简洁的文字给人物设置了一个道德困境。

  D.小说的结尾照应了题目,既出人意料,又在情理之中;而且,使得王老汉“诚信”的形象更为突出。

  8.小说中两处写到“雪”,分别有何作用?请结合文本简析。(5分)

  9.小说中王老汉和吴亚龙,哪一位是主人公?请结合文本谈谈你的看法。(6分)

  二、古代诗文阅读(35分)

  (一)文言文阅读(19分)

  阅读下面的文言文,完成10—13题。

  周起字万卿,淄州邹*人。生而丰下,父意异之,曰:“此儿必起吾门。”因名起。幼敏慧如*。意知卫州,坐事削官,起才十三,诣京师讼父冤,父乃得复故官。举进士,授将作监丞。擢著作佐郎,累迁户部度支判官。真宗北征,领随军粮草事,寻为东京留守判官,判登闻鼓院。以右正言知制诰,权判吏部流内铨。封泰山,摄御史中丞,所过得采访官吏能否及民利病以闻。东封还,近臣率颂功德,起独以居安为戒。初置纠察刑狱司,因命起,起乃请诸已决而事有所枉及官吏非理榜掠者,并听受诉,从之。擢枢密直学士、权知开封府。起听断明审,举无留事。真宗尝临幸问劳,起请曰:“陛下昔龙潜于此,请避正寝,居西庑。”诏从之,名其堂曰继照。起尝奏事殿中,适仁宗始生,帝曰:“卿知朕喜乎?宜贺我有子矣。”即入禁中,怀金钱出,探以赐起。从祀汾阴,权知河中府,徙永兴、天雄军,所至有风烈,数赐书褒谕。拜给事中,进礼部侍郎。起素善寇准,尝与寇准过同列曹玮家饮酒,既而客多引去者,独起与寇准尽醉,夜漏上乃归。明日入见,引咎伏谢,真宗笑曰:“天下无事,大臣相与饮酒,何过之有?”准且贬,起亦罢为户部郎中,又降太常少卿。后复为礼部侍郎,以疾请知颖州,徙陈州。卒,赠礼部尚书,谥安惠。起性周密,凡奏事及答禁中语,随辄焚草,故其言,外人无知者。家藏书至万余卷。起能书。弟超,亦能书,集古今人书并所更体法,为《书苑》十卷。

  (节选自《宋史•列传第四十七》)

  10.对下列句子中加点字的解释,不正确的一项是(3分)

  A.生而丰下,父意异之 认为……奇特

  B.摄御史中丞 代理

  C.擢枢密直学士、权知开封府 暂且

  D.起素善寇准 好好地

  11.下列对文中加点词语的相关内容的解说,不正确的一项是(3分)

  A.庙号是古代帝王、大臣等死后,据其生*事迹评定的称号,如真宗、仁宗。

  B.泰山封禅起源于春秋战国时期,是帝王受命于天的典礼,目的是巩固皇权,粉饰太*。

  C.礼部为六部之一,掌管礼仪、祭祀、学校、科举等,长官为礼部尚书。

  D.我国古代以铜壶滴漏计算时间,夜漏就是深夜时分。

  12.下列对原文有关内容的概括和分析,不正确的一项是(3分)

  A.周起关注民生,思虑长远。他把采访官吏能力大小及百姓疾苦报知朝廷;东封还朝后,皇上近臣都歌功颂德,唯有周起认为应该居安思危。

  B.周起善于断案,明辨是非。担任纠察刑狱司官员时,周起倾听了那些已经判决但事实确有冤情,以及官吏胡乱拷打的犯人的申诉,受到皇上慰劳。

  C.周起交好寇准,受到牵连。他一向与寇准关系好,曾到寇准家喝酒,尽醉方归;寇准将要被贬官,周起也被连续降职。

  D.周起为人谨慎,做事周密。凡是上奏事情以及书面应答宫中的文书,周起随后就烧掉草稿,他的言语看法,外人没有知晓的。

  13.把文中画横线的句子翻译成现代汉语。(10分)

  (1)意知卫州,坐事削官,起才十三,诣京师讼父冤,父乃得复故官。

  (2)从祀汾阴,权知河中府,徙永兴、天雄军,所至有风烈,数赐书褒谕。

  (二)古代诗歌阅读(11分)

  阅读下面这首宋词,完成14-15题。

  西江月①

  【宋】苏轼

  照野弥弥②浅浪,横空隐隐层霄③。障泥④未解玉骢骄,我欲醉眠芳草。

  可惜一溪风月,莫教踏碎琼瑶⑤。解鞍欹枕绿扬桥,杜宇一声春晓。

  【注】①此词为苏轼被贬黄州作。其春夜行蕲水边,过酒家,饮醉,乘月至一溪桥上,解鞍,枕臂醉卧少休。拂晓,见乱山攒拥,流水淙淙,宛如仙境,书此词于桥柱上。②弥弥:水波翻动的样子。③层霄:弥漫的云气。④障泥:马鞯,垂于马两侧以挡泥土。⑤琼瑶:美玉。这里形容月亮在水中的倒影。

  14.下列对本词的理解,不正确的两项是(5分)

  A.“障泥未解玉骢骄,我欲醉眠芳草”,写词人频临溪流,从马上下来,等不及卸下马鞯,即欲眠于芳草。既侧面描绘出月下溪景之美,又表现了词人的喜悦心情。

  B.“可惜一溪风月,莫教踏碎琼瑶”一句运用了借代的修辞手法。琼瑶,本是美玉,这里代指月色,水月交辉,有如晶莹剔透的珠玉。此句传神地写出水月之静美。

  C.“解鞍欹枕绿杨桥,杜宇一声春晓”一句用“解鞍欹枕”这个特写镜头表现出词人的随意豁达,而“杜鹃春晓”写出空山春晨的喧闹和生机。

  D.本词写作者爱惜一溪风月,不让马踏碎溪月,解鞍酣眠直至春晓,了一个醉心自然、随性洒脱、乐观豁达的诗人形象。

  15.请从表达手法的角度简要赏析“照野弥弥浅浪,横空隐隐层霄”两句的妙处。(6分)

  (三)名篇名句默写(5分)

  16.补写出下列句子中的空缺部分。(5分)

  (1)在《念奴娇•赤壁怀古》中词人神游故国后,用“ , ”来书写自己多愁善感而顿觉年华逝去的感叹。

  (2)在《生于忧患死于安乐》中,孟子认为上天要让一个人担负重大使命,一定会让这个人经历种种艰难困苦,最终“ , 。”

  (3)在《永遇乐•京口北固亭怀古》中,辛弃疾回顾了元嘉年间的那次北伐,宋文帝刘义隆本希望能够“封狼居胥”,但是由于行事草率,最终却“ ”。

  第Ⅱ卷 表达题(80分)

  三、语言文字运用(20分)

  17.下列各句中加点成语的使用,全都正确的一项是(3分)

  ①从历史上看,人类社会每一个新时代的开启,似乎都与技术变革息息相关。如今,正是新的技术革命爆发的前夜。

  ②他心高气傲,目空一切,总喜欢妄自菲薄别人,结果可想而知,没有人愿意跟他打交道,他成了大海里的一叶孤舟。

  ③在义工联盟中,他不仅是发起人、带头者,更是义工精神的实践者、坚守者,时时处处都处心积虑,亲力亲为。

  ④82岁高龄的袁隆*面对辉煌没有任何改变,他依然风尘仆仆地骑着摩托车去实验田,依然从北到南查看育种基地。

  ⑤莫言小说最先征服你的并不是故事和人物,而是语言。那一个个*淡的文字背后深藏着穿

  云裂石的哀痛和精彩斑斓的怜爱。

  ⑥自2015年6月起,经过三个月的改造,仁和花园焕然一新。如今,小区活动广场内篮球场、健身器材、桌椅等一应俱全。

  A.①④⑥ B.②③⑥ C.①④⑤ D.③⑤⑥

  18.下列各句中,没有语病的一句是(3分)

  A.峰会期间新闻中心共接待大约来自近70个国家中外媒体记者共1.8万人次,并成功举办12场新闻发布会。

  B.在里约残奥会上,中国残奥代表团共获107枚金牌、81枚银牌和51枚铜牌,239块奖牌的骄人成绩连续第四次占据金牌榜与奖牌榜双第一。

  C.根据国家统计局9月9日发布的数据显示,8月份CPI(全国居民消费价格总水*)同比上涨1.3%,创下今年以来新低。

  D.日前,北京市公布了《北京市关于进一步推进户籍制度改革的实施意见》。至此,全国已出台户籍制度改革方案的省份达到30个。

  19.填入下面文段空白处的词语,最恰当的一组是 (3分)

  爱因斯坦的左脑特别发达,__①__他总是使用左脑。他分析自然现象的能力和形成观念的能力非常卓越。__②__有趣的是,他__③__用语言__④__用非语言(比如图形)进行思考的,__⑤__是以一种跳跃的方式。完成之后,再将它转换成语言,__⑥__从左脑转到右脑。这就是“思维错位症”。

  ① ② ③ ④ ⑤ ⑥

  A 因为 但 不是 而是 而且 即

  B /[ 并且 不仅 而且 就是 即

  C 但是 但 不是 而是 并且 然后

  D 因为 并且 不仅 而且 就是 然后

  20.在下面一段文字横线处补写恰当的语句,使整段文字语意完整连贯,内容贴切,逻辑严密。每处不超过15个字。(5分)

  “字”是我们工作及生活中必不可少的部分,那么,① ?关于汉字的起源,一直是我国学者研究的对象,但一直② ,直到最近几十年,我国考古学家才先后发布了一系列“安阳殷墟甲骨文”及与汉字起源有关的出土资料。资料显示,我国最早的汉字可追溯到原始社会晚期及人类社会早期在陶器上面刻画或彩绘出的符号,同时,还有少量刻写在甲骨、玉器、石器等上面的符号。这些符号,距今已经有8000多年历史。由此可见,③ 。

  21.下面是某宾馆住客控制流程图,请把这个构思写成一段话,要求内容完整,表达准确,语言连贯,不超过60字。(5分)

  四、写作(60分)

  22.阅读下面的材料,根据要求写一篇不少于800字的文章。(60分)

  林丹和李宗伟,一个是金牌拿到手软,光世界冠军就有将近20个的大满贯得主;一个是穷其一生可能也无法实现奥运冠军梦的“千年老二”。我们会记得林丹夺冠后的疯狂,也有不少人忘不了李宗伟落寞的身影。有网友说,若没有李宗伟,林丹纵然再强,也无法被激发出最好的自己。林丹说:“你们除了看到我以外,也应该看到我的对手李宗伟,我不觉得他是

  失败的。他通过努力让很多人也认可他,我觉得这就足够了。金牌只是一种标准,不能够代表所有。有时我也会向李宗伟学习,我会想他输给了我这么多次重要比赛,为什么还能够放下一切,继续跟我再拼下一次比赛?”李宗伟在接受采访时说:“其实冠军对我来说已经不是很重要了,我想的是接下来看能不能再多打两年,慢慢享受羽毛球吧。”

  要求:选好角度,确定立意,明确文体,自拟标题;不要脱离材料内容及含意的范围文,不要套作,不得抄袭。


英语高考北京模拟试卷及答案(扩展2)

——届山东桓台高考英语模拟试卷及答案

届山东桓台高考英语模拟试卷及答案1

  第一部分 听力(共两节,满分30分)

  第一节 (共5小题;每小题1.5分,满分7.5分)

  听下面5段对话。每段对话后有一个小题,从题中所给的A、B、C三个选项中选出最佳选项,并标在试卷的相应位置。听完每段对话后,你都有10秒钟的时间来回答有关小题和阅读下一小题。每段对话仅读一遍。

  1. What time is it now?

  A. 6: 40. B. 6: 30. C. 6: 20.

  2. What does the man mean?

  A. They have left for the airport.

  B. They are on the way to the airport.

  C. They may be late for the plane.

  3. What kind of music does the woman like?

  A. Popular music. B. Jazz music. C. Classical music.

  4. What does the man suggest the woman do?

  A. Take some medicine. B. Drink more water. C. Go on a diet.

  5. What does the man tell the woman?

  A. There is another cat like his.

  B. He never loses his dog at all.

  C. She has mistaken it for his dog.

  第二节 (共15小题;每小题1.5分,满分22.5分)

  听下面5段对话或独白。每段对话或独白后有几个小题,从题中所给的A、B、C三个选项中选出最佳选项,并标在试卷的相应位置。听每段对话或独白前,你将有时间阅读各个小题,每小题5秒钟;听完后,各小题将给出 5秒钟的作答时间。每段对话或独白读两遍。

  听第6段材料,回答第6、7题。

  6. How soon will the man finish his book?

  A. In several months. B. In a few days. C. In two months.

  7. What is the man going to do?

  A. He will market his book himself.

  B. He will ask his friends to advertise the book.

  C. He hasn’t made a decision.

  听第7 段材料,回答第8.9 题

  8. When is Alice’s birthday?

  A. The next day. B. The day after tomorrow. C. The day they had the talk.

  9. What will the man and the woman buy for Alice?

  A. A record. B. Some flowers. C. A box of chocolates.

  听第8 段材料,回答第10至12题

  10. What does the woman say about the restaurant?

  A. It is very old. B. The fish there is fresh. C. The beef there is delicious.

  11. What do we know about the restaurant?

  A. It serves roast dinners on Sundays.

  B. It doesn’t serve vegetarian dishes.

  C. It opens from 7: 00 p. m. to 11: 00 p. m. on Sundays.

  12. What will the man do next?

  A. Book a table. B. Check the menu. C. Call the woman’s sister.

  听第9 段材料,回答第13至16题

  13. What are the speakers talking about?

  A. Their own parents. B. Their own changes. C. Their long-lost relatives.

  14. What does the man say about Aunt Gertrude?

  A. She is looking so old. B. She is so thin now. C. She has long hair.

  15. What does Cousin Emily wear?

  A. An orange dress. B. A yellow dress. C. A white dress.

  16. Where does Aunt Jane live now?

  A. In Boston. B. In Chicago. C. In Atlantic City.

  听第10段材料,回答第17至20题

  17. How will the adventurers cross the Amazon River?

  A. By train. B. By boat. C. By bike.

  18. What will the adventurers do in the rain forests of Brazil?

  A. See the sunrise.

  B. Look at a variety of animals.

  C. Take a boat to the base of the falls.

  19. Where will the adventurers go in the second week?

  A. To the Mount Fuji. B. To the Niagara Falls. C. To the Amazon River.

  20. What does the speaker say about the temples in Thailand?

  A. They are incredible. B. They are unforgettable. C. They are indescribable.

  第二部分 阅读理解(共两节,满分40分)

  第一节 (共15小题;每小题2分,满分30分)

  阅读下列短文,从每题所给的四个选项(A、B、C和D)中,选出最佳选项。并在答题卡上将该选项涂黑.

  A

  Sit down,close your eyes and relax. Think about your childhood and the memories you created. What was your favorite childhood memory and what was your favorite as a child? Or what your favorite Christmas present or your favorite candy bar when growing up? Answers may come easily for some people but for others,it may take more time to think about.

  Mary Jo McCarthy of Pequot Lakes wants to help others document their childhood memories which could be treasured by current families and for generations to come. McCarthy, 64, a short story author and retired columnist(专栏作家)for the Lake Country Echo weekly newspaper, began hosting a writing seminar(研讨班)titled“A Time to Remember”.The classes are limited to 12 people and are conducted in the nursery at Lutheran Church of the Cross in Nisswa and the registration fee is $25.

  McCarthy said what led her to start the writing seminars was writing others’ biographies. A retired businessman asked her to write his biography as a ghost writer. She said others from her church in Pequot Lakes had also asked her to write their biographies because they didn’t know how to write them. She thought hosting a class to help people write their own biographies would be more beneficial to them.

  McCarthy said people had busy lifestyles those days and didn’t have a chance to sit down and write down their memories that they might forget and never remember. She said writing down the memories would document the person’s life,go other family members,especially future generations would be able to read about them.

  21.In paragraph l the author raises some questions to________

  A.introduce the topic of the text

  B.express his/her puzzle about memories

  C.give some examples for readers about biographies

  D.provide background details for readers about the retired columnist

  22.What can we learn about McCarthy?

  A.She was strict with her students

  B.She didn’t take charge of seminars

  C.She got support from the local government

  D.She once worked as a columnist in a newspaper.

  23. You are very likely to read the passage in ______.

  A. a biology textbook B. A health report C. a newspaper D. a book review

  24.The passage is mainly intended to________.

  A.show how to write biographies

  B.help people communicate with their families

  C.call on people to write their own biographies

  D.introduce McCarthy and her writing seminars

  B

  Recently I spoke to some of my students about what they wanted to do after they graduated, and what kind of job prospects they thought they had.

  Given that I teach students who are trained to be doctors, I was surprised to find that most thought that they would not be able to get the jobs they wanted without“outside help”. “What kind of help is that? ”I asked, expecting them to tell me that they would need a relative or family friend to help them out.

  “Surgery(外科手术), ”one replied. I was pretty alarmed by that response. It seems that the graduates of today are increasingly willing to go under the knife to get ahead of others when it comes to getting a job. One girl told me that she was considering surgery to increase her height. “They break your legs, put in special extending screws, and slowly expand the gap between the two ends of the bone as it regrows. You can get at least 5 cm taller! ”

  At that point, I was shocked. I am short. I can’t deny that, but I don’t think I would put myself through months of agony(痛苦)just to be a few centimeters taller. I don’t even bother to wear shoes with thick soles, as I’m not trying to hide the fact that I am just not tall!

  It seems to me that there is a trend toward wanting“perfection”, and that is an ideal that just does not exist in reality.

  No one is born perfect, yet magazines, TV shows and movies present images of thin, tall, beautiful people as being the norm. Advertisements for slimming aids, beauty treatments and cosmetic surgery clinics fill the pages of newspapers, further creating an idea that“perfection”is a requirement, and that it must be purchased, no matter what the cost is. In my opinion, skills, rather than appearance, should determine how successful a person is in his chosen career.

  25.Many graduates today turn to cosmetic surgery(美容手术) to   .

  A. marry a better man/woman

  B. become a model

  C. get an advantage over others in job-hunting

  D. attract more admirers

  26. According to the passage, the author believes that   .

  A. everyone should pursue perfection, whatever the cost is

  B. it’s right for graduates to ask for others to help them out in hunting for jobs

  C. media are to blame for misleading young people in their seeking for surgery

  D. it is one’s appearance instead of skills that really * in one’s career

  27. What does the author think of his height?

  A. He hates to be called a short man.

  B. He tries to increase his height through surgery.

  C. He always wears shoes with thick soles to hide the fact.

  D. He just accepts it as it is.

  C

  Can eating a chocolate bar every day really prevent age-related memory loss? No. But a new research shows that large amounts of flavones, substances found in cocoa, tea and some kinds of vegetables, may improve age-related memory failure.

  Dr. Scott Small is a professor of neurology (神经病学) at Columbia University in new York City. He is the lead writer of a research paper describing the effects of cocoa flavones on brain activity. His study involved 37 volunteers aged between 50 and 69. Researchers gave them a high-level flavones drink made from cocoa beans or a low-level flavones drink. For a period of three months, some subjects got 900 milligrams of flavones a day. The others got 10 milligrams of flavones each day. Brain imaging and memory tests were given to each study subject before and after the study. Dr. Small says that the subjects who had the high-level flavones drink showed much improvement on memory tests.

  The researchers warn that more work is needed to be done because this study was performed only on a small group. Dr. Joann Manson is the lead researcher of a four-year study involving 18,000 adults. This study will use flavones capsules(胶囊). The study subjects will be divided into two groups and will take two pills per day. The capsules used will all look the same. But one group"s capsules will contain flavones, while the other group will take capsules made of an inactive substance, or placebo

  Dr. Manson says it"s not necessary for people to start eating more chocolate, because a person would have to eat a huge amount of chocolate to get the same level of flavones given to the rest subjects. He adds many manufacturers have planned to remove the flavones from their chocolate products. Similarly, Dr. Manson says a cocoa-based flavones extract(提取)may be developed in the future But he says that more studies are needed to see how much flavones is good for our health.

  28. What was done to the subjects after Dr. Small"s study?

  A. They were asked to take a high-level flavones drink.

  B. They were given capsules containing flavones.

  C. They were given brain imaging and memory tests.

  D. They were asked to take two pills of flavones capsules per day.

  29. What"s the similarity of the two experiments?

  A. Both use high-level flavones capsules.

  B. The number of the subjects is the same.

  C. Drinks and placebos are used in both experiments.

  D. The subjects are divided into two groups in both experiments.

  30. Why will Dr. Manson carry out the four-year study?

  A. To prove the first experiment is wrong.

  B. To carry out the experiment further.

  C. To test how much flavones can improve our memory.

  D. To show eating chocolate is better to improve memory.

  31. What can we learn from the last paragraph?

  A It"s not necessary for us to eat chocolate.

  B The more we eat flavones, the better our health will be.

  C. In the future we can get flavones without eating chocolate.

  D. It"s easy for people to get the same level of flavones given to the test subjects.

  D

  There has been much debate since Titanic became one of the most successful films of all time: Did Rose leave Jack to die by not letting him up on the makeshift lifeboat as they floated in the ocean?

  A U.S. television show, the Discovery Channel’s Mythbusters recreated the dangerous conditions after the Titanic sank in 1912, and through a chain of hypothermia(体温过低) tests, has discovered Leonardo DiCaprio’s character would have survived the disaster had he lifted himself onto the board from which Rose, played by Kate Winslet, was rescued.

  As fans of the film remember, Jack remains in the freezing water while Rose lies on a floating piece of the destroyed ship as he doesn’t want it turned over.

  In the Oscar-winning film, Jack dies from hypothermia while Rose is rescued. Titanic director James Cameron told Mythbusters he believed the board wouldn’t have supported both Jack and Rose.

  But the team found if Rose had taken off her life jacket and put it under the board it would have given them enough buoyancy to keep both of them floating.

  In the film Rose was one of the last to be rescued. Historical documents show that the Officer Harold Lowe, who was made to rescue Rose, looked for survivors for 45 minutes.

  In a separate test the team found that had Jack remained in the freezing water for over an hour he would have died from hypothermia.

  Cameron, however, had the last word telling the TV show that Jack had to die as that’s what the plot called for. He said, “I think you’re missing the point here. The plot says Jack died. He has to die. So maybe we should have made the board a tiny bit smaller, but he’s still going down.”

  32. The U.S. TV show did tests on Jack’s death ______.

  A. after creating a difficult condition

  B. by tying Rose’s life jacket to the boat

  C. by testing how long one can hold out in cold water

  D. after talking to the US director James Cameron

  33. What does the underlined word “buoyancy” in Paragraph 5 probably mean?

  A. Space. B. Warmth. C. Softness. D. Force.

  34. Which of the following is TRUE according to the text?

  A. Finally Cameron agreed with the team.

  B. Rose was the last one to be rescued.

  C. Cameron thought the board was hard enough.

  D. Kate Winslet’s character died from hypothermia.

  35. What would be the best title for the text?

  A. A superb film Titanic B. Jack’s Death for Nothing

  C. Cameron’s Unreasonable Argument D. Jack’s Deep Love with Kate Winslet

  第二节(共5小题;每小题2分,满分10分)

  根据短文内容,从短文后的选项中选出能填入空白处的最佳选项。选项中有两项为多余选项。

  How to communicate effectively

  Effective communication is a must for everyone who hopes to be successful. It helps express your thoughts and improves your career prospects (前景) as well as strengthens relationships. Therefore, it’s necessary to work hard to improve your communication skills.

  36

  Before you are ready to talk to a group of people, you should think about what you are going to talk about. Although a printed copy of a speech may not be always necessary, highlighting a few important issues in your mind is a must. 37 And it reduces the possibilities of getting away from the main issues.

  Choose your words

  Choice of words is the most important part of any communication. Simple, clear words are appropriate for the issue you are discussing. Needless to say, words that could make your audience unpleasant are best kept out of the communication. Most people tend to be careful about their language while at work. 38 This may hurt your loved ones beyond your imagination.

  Body language

  Many people use body language by mistake and end up making a fool of themselves. Body language means keeping eye contact and using hand gestures only wherever required. 39 This will make your talk more of a stage performance and defeat its purpose.

  Listen and concentrate

  Communication is an exchange of ideas. Once you are done with your part, listen carefully to what other people plan to say. 40 Get rid of all distractions (分散注意力的事物) such as a phone before you begin communication. A distraction not only disturbs your focus, but it also causes your audience’s attention to wander.

  A. Gather your thoughts

  B. Prepare a copy of a speech

  C. However, when they get home they often use colorful language.

  D. This will help you remain focused throughout your communication.

  E. If you are giving a speech, it is important that you focus on that activity only.

  F. Effective communication is a gradual process and requires efforts on your part.

  G. Remember, there is no need to speak every word with a gesture or facial expression.

  第三部分 英语知识运用(共两节,满分45分)

  第一节 完形填空(共20小题;每题1.5分,满分30分)

  阅读下面短文,从短文后各题所给的四个选项(A, B, C和D)中,选出可以填入空白处的最佳选项,并在答题卡上将该项涂黑。

  When I was growing up my dad would often give me small tasks to finish to help out around the house. Some were easy but the three I remember the most were the three 41 of all. The first was helping to weed the four 42 we had. It was 43 work. I would rather ride my bike than crawl(爬行)in the dirt on my hands and knees 44 the thousands of weeds in our gardens. The second was piling firewood for 45 . It was tough work and I got more than my 46 of pieces. I would rather walk in the woods than piling it in rows. The third was 47 water from a mountain spring to our house. 48 our well water had too much iron in it to 49 .

  Over the years, though, I learned to see the 50 in all the things my dad had made me do. Having fresh vegetables for dinner was a delight. 51 a cold glass of iced ten made from the mountain spring water was a pure 52 . Standing by the wood stove on a snowy December day was a wonderful way to 53 . I saw as well that all this work my dad had given me had

  54 made me a better, stronger and more caring man. Most of all, I 55 that when I did my work with a(n) 56 heart, it didn’t fell like work at all.

  Over the years I have learned something else too. When we do the work for our Heavenly Father with a 57 spirit, it doesn’t feel like work, either. It feels like every act of kindness, word of encouragement, and gift of love we share will only make our lives better, make our hearts happier, and bring out souls 58 to Heaven.

  I am so 59 to both my dad and my Father in Heaven. They showed me the 60 of work and more importantly they showed me the pricelessness of love.

  41. A. simplest B. toughest C. strangest D. wildest

  42. A. schools B. companies C. gardens D. streets

  43. A. boring B. exciting C. interesting D. relaxing

  44. A. controlling B. keeping C. pushing D. pulling

  45. A. festival B. winter C. holiday D. marriage

  46. A. share B. charge C. point D. dream

  47. A. protecting B. borrowing C. carrying D. conducing

  48. A. because B. unless C. although D. but

  49. A. boil B. match C. get D. drink

  50. A. requirements B. attention C. rewards D. experience

  51. A. Designing B. Enjoying C. Performing D. Developing

  52. A. desire B. challenge C. surprise D. pleasure

  53. A. turn up B. warm up C. stay up D. set up

  54. A. unluckily B. immediately C. gradually D. temporarily

  55. A. remembered B. ignored C. regretted D. realized

  56. A. honest B. happy C. brave D. modest

  57. A. loving B. complaining C. hating D. punishing

  58. A. closer B. farther C. politer D. warmer

  59. A. kind B. lucky C. grateful D. vital

  60. A. key B. way C. goal D. value

  第Ⅱ卷(共50分)

  注意事项:

  第Ⅱ卷共2页。考生必须使用0.5毫米黑色签字笔在答题卡上各题目的指定答题区域内作答,在试卷上作答无效。

  第二节 (共10题;每小题1.5分,满分15分)

  阅读下面材料,在空白处填入适当的内容(1个单词)或括号内单词的正确形式。

  A recent survey shows that storytelling ranks before television or video games among pastimes(消遣)for kids. The survey was carried out among 500 children aged 3—4 in Britain, of __61 over three—quarters said they wished their parents could read to them more often. Almost two—thirds of the kids said reading a story with their parents helped them sleep 62 (well) and more easily. According to the survey, there are two ways for parents 63 (become) good storytellers. They can use 64 (fun) voices to show different characters in a story. They can also make their own special sounds to keep the story 65 (interest) and attractive. When Mum and Dad aren’t at hand, over 30% of kids would like to hear 66 bedtime story from famous people like Harry Potter star Daniel Radcliffe. Other movie stars like Zac Efron, Miley Cyrus and Emma Watson 67 (welcome)by children, too.

  “ 68 can be very difficult for parents to find the time to read with their children. But these moments can help build close relationship 69 parents and children, and also can play an important part in their child’s 70 (develop),” said child psychologist Woolfson.

  Whom,

  第四部分:写作(共两节,满分35分)

  第一节短文改错(共10小题;每小题1分,满分10分)

  假定英语课上老师要求同桌之间交换修改作文,请你修改你同桌写的.以下作文。文*

  有10处错误,每句中最多有两处。错误涉及一个单词的增加、删除或修改。

  增加:在缺词处加一个漏字符号(Λ),并在其下面写出该加的词。

  删除:把多余的词用斜线(\)划掉。

  修改:在错的词下划一横线,并在该词下面写出修改后的词。

  注意:

  1.每处错误及其修改均仅限一词;

  2.只允许修改10处,多者(从第11处起)不计分。

  It was a freezing winter night. I pulled myself along the way before a long day’s work standing behind the counter. With an empty stomach and only ten-dollar note and a few pennies in the pocket, my life seemed hopelessly. Despite the warm Christmas atmosphere, my heart was frozen to dead. “Excuse me, Madam. Would you please be too kind as to give a few pennies? The little angels will appreciate for it.” A sweet voice rise behind me. I turned and found a plain-looked girl with two abandoned dogs. Without hesitations, I handed over the note in my pocket. Cheerfully the girl went away and she would never know how she saved my day and my life.

  第二节 书面表达 (25分)

  假设你是某外国语学校的学生会*。最近,为了弘扬雷锋精神,你校决定开展多项志愿服务活动。请用英语写一篇倡议书,发表在校英语报上,号召同学们加入本校的志愿者俱乐部。内容包括:

  1.陈述当志愿者的好处;

  2.向同学们发出几点倡议。

  注意:

  1.词数120左右;

  2.可以适当增加细节,以使行文连贯;

  3.开头已给出,但不计入总词数。

  Dear fellow students,

  Volunteering has a positive effect on our community. ______________________________________

  _______________________________________________________________________

  _______________________________________________________________________

  Students’ Union


英语高考北京模拟试卷及答案(扩展3)

——届河北高考英语模拟试卷及答案

届河北高考英语模拟试卷及答案1

  第一部分:听力(共两节,满分30分)

  第一节(共5小题;每小题1.5分,满分7.5分)

  听下面5段对话,每段对话后有一个小题,从题中所给的A、B、C三个选项中选出最佳选项,并标在试卷的相应位置。听完每段对话后,你都有10秒钟的时间来回答有关小题和阅读下一小题。每段对话仅读一遍。

  1. What are the speakers talking about?

  A. Mending cameras. B. Painting pictures. C. Taking photographs.

  2. Where does the conversation take place?

  A. At a hotel. B. At a library. C. At a post office.

  3. How does the woman feel about the acting?

  A. It isn’t good. B. She loves it. C. It’s just so-so.

  4. What is the boy complaining about?

  A. His new job. B. At a tennis court. C. Yard work.

  5. What will the speaker do next?

  A. Move the old man.

  B. Call for help.

  C. Report to the police

  第二节(共15小题;每小题1.5分,满分22.5分)

  听下面5段对话或独白。每段对话或独白后有几个小题,从题中所给的A、B、C三个选项中选出最佳选项,并标在试卷的相应位置。听每段对话或独白前,你将有时间阅读各个小题,每小题5秒钟;听完后,各小题将给出5秒钟的作答时间。每段对话或独白读两遍。

  请听第6段材料,回答第6和第7题。

  6.How much time does the woman have to go to meet the man ?

  A. 30 minutes. B. 45 minutes. C. 55 minutes

  7.How will the speakers go to the Smiths"?

  A. Ride in a friend’s car. B. Go by bus. C. Go by bike.

  请听第7段材料,回答第8至9题。

  8. Where does the conversation take place?

  A. At a clinic. B. At a restaurant. C. At a pharmacy.

  9. What can we learn about the woman?

  A. She’s allergic to seafood.

  B. She ate too much.

  C. She ran too much.

  请听第8段材料,回答第10至12题。

  10. What is the probable relationship between the speakers?

  A. Schoolmates. B. Colleagues. C. Neighbors.

  11. Where are the speakers?

  A. In the US. B. In Vietnam. C. In Italy.

  12. What is the woman job?

  A. A college lecturer.

  B. An assistant of a nurse.

  C. An engine mechanic.

  请听第9段材料,回答第13至16题。

  13. Where are the speakers?

  A. At a concert hall. B. At a snack bar. C. At a movie house.

  14. What will the woman do next?

  A. Get some snacks.

  B. Get two better seats.

  C. Go over her lessons.

  15. What does the woman think of her last experience?

  A. It was terrific.

  B. It was terrible.

  C. It was interesting.

  16. What can we learn from the conversation?

  A. Many people are at the opening.

  B. The man is over-weight.

  C. The woman is careless.

  请听第10段材料,回答第17至20题。

  17. What caused the fire?

  A. An irresponsible driver.

  B. The dry weather.

  C. A lightning.

  18. In which direction of Stanton is the lake?

  A. The southwest. B. The northwest. C. The west.

  19. Who reported the serious situation to the chief fire officer?

  A. A truck driver. B. The motorist. C. A fireman.

  20. How did the chief officer decide to stop the fire at last?

  A. Clear an area in the forest.

  B. Send more firemen.

  C. Blow down the burning trees.

  第二部分 阅读理解(共两节,满分40分)

  第一节 (共15小题; 每小题2分,满分30分)

  阅读下列短文,从每题所给的四个选项中,选出最佳选项,并在答题卡上将该项涂黑。

  A

  Want to help your little kids develop better physically and mentally? My Gym has some programs for you.

  Little Bundles (6 weeks-6 months)

  Our youngest My Gym students begin their first visual, audile (听觉的)and spatial exploration while they and their parents are guided through the program. The program is designed to carefully introduce music and movement in various fun ways. Baby exercises, songs, dances, baby-safe rides, unique swings, and many other Parent & Me adventures await the baby-parent team. Each week parents participate in a dialogue about their baby’s development.

  Tiny Tykes (7-13 months)

  As the babies gain hand, arm and body control and become mobile, we introduce new activities. Our babies and their parents are led through a variety of beginning stretches (伸展),exercises, baby songs and dances, baby-safe rides and adventures, and balance skills during this program. A relationship to movement and physical activity begins to develop, as our babies discover their natural abilities and improve their physical and cognitive (认知的) development.

  Waddlers (14-22 months)

  At this age, a kid’s level of independent mobility typically progress from aggressive crawling to running. Additional motor skills developing during this stage include kicking a ball, building a block tower, riding a tricycle, and walking up stairs. Our highly trained instructors shower the children with love and caring, as they guide our kids in early social skills through interactive songs, well-designed dances, age-specific puppet (木偶)shows, special rides, and a variety of swings and other original activities.

  Gymsters (23 months-2 l/2 years)

  During this program, the youngsters are happy as they sing, dance, swing, play games, take turns on special rides, and have “Surprise Time”. Our kids feel great about themselves as they learn gymnastics: in a constantly changing program with fresh activities and new equipment each week.

  21. What does Little Bundles require parents to do once a week?

  A. Take an adventurous trip.

  B. Design a program for kids.

  C. Talk about their kid’s growth.

  D. Share learning skills with others.

  22. In which program can babies watch puppet shows?

  A. Little Bundles. B. Tiny Tykes.

  C. Waddlers. D. Gymsters.

  23. What can we learn about Gymsters?

  A. It isn’t as fun as the other three programs.

  B. Kids can do new activities every week.

  C. It requires plenty of parents’ participation.

  D. The activities it involves remain unknown.

  24. What is the purpose of this text?

  A. To advertise some programs at My Gym.

  B. To show what activities kids are interested in.

  C. To tell parents how to help their kids develop.

  D. To encourage readers to do exercise at My Gym.

  B

  Two teachers are sitting in a laboratory, their dirty hands buried in grass. Mary Richmond is a sixth grade science teacher at Cache LaPoudre Middle School in Laporte, Colo. Jolene McDowell teaches high school biology at Great Mills High School in Maryland. The two women have spent the past week doing science as part of the Research Experience for Teachers Program at the Toolik Field Station, only 188 kilometers south of the Arctic Ocean. Armed with the research experience, they will be excited to head back to share their stories with their students about science in the Arctic in a few weeks.

  For their two-week experience, Richmond and McDowell are living in a tent. So far, they have helped to set up a laboratory, processed soil samples and mounted other samples onto microscope slides.

  “As teachers in the classroom, I think it’s important for us to get out and do science, Richmond says. This experience has rekindled (重新点燃)her enthusiasm for science, she says. Richmond is looking forward to taking it back to the classroom. “If I’m excited about it, the kids are more likely to be excited about it,’’ she notes. And in her sixth grade classroom, enthusiasm is important.

  “It’s also important to show students that you want to learn and you’ re willing to put yourself out there and be uncomfortable,” explains McDowell. “I’ve never been this far north before. This is a location where you don’t even flush (冲)your toilet paper. But I’m willing to be here to learn and experience something new,” she adds.

  The Toolik research station brings in a different number of teachers each year. Openings are generally posted in March and can be found on the Natural Resource Ecology Laboratory website.

  After their stay, the two teachers, along with others at research sites around the United States, will have a year to translate their experience to something for the classroom. They can add to their curriculum, make a presentation or come up with a class activity.

  25. Why are the two teachers at the Toolik Field Station?

  A. They’ve been there doing science.

  B. They’ve been invited to hold lectures.

  C. They’re planning to set up a laboratory.

  D. They’re volunteering to experience life.

  26. What do the two teachers probably think of their two-week experience?

  A. Relaxing and exciting.

  B. Challenging but useless.

  C. Dangerous but beneficial.

  D. Unforgettable and beneficial.

  27. What would be the best title for the text?

  A. Teachers teach science at the Toolik Field Station

  B. Teachers inspire students to do science in class

  C. Teachers experience difficulties in the Arctic

  D. Teachers get to do cool science in the Arctic

  C

  Chocolate is the “food of the gods,’’ a sweet treat for many across the world, and an increasingly popular industry worth an estimated $110 billion a year. But chocolate lovers, your beloved snack may have just been saved by another sweet treat: mangoes (芒果).Scientists may have found a way to solve a possibly coming shortage of cocoa, which could affect future chocolate production, by using mangoes in place of cocoa to make chocolate, according to a study. “Wild mango is one of the so-called Cinderella (灰姑娘) species whose real value is unrealized,’’ says Akhter, the study’s senior author.

  Global cocoa production has gone down in recent years — due to a handful of factors including changes in climate and crop failure — while the demand for cocoa has been on the rise. Cocoa producers in the past also have been accused of unfair labor practices, including employing child laborers and underpaying farmers. Wild mango butter may be chemically and physically similar enough to cocoa butter to act as a replacement, which makes researchers and food producers excited and hopeful.

  The study’s authors also believe the potential business benefits of the fruit could be a boon to protection efforts. “Going beyond the use to industry, wild fruits like mangoes are an important source of food, medicine and income for rural people, but are in decline due to factors such as deforestation,’’ said Morag McDonald of Bangor University, another of the study’s authors. Adding value to underused products through processing for products that have market value can bring about a valuable motivation for the conservation of such species, and help to bring about alternative income sources and reduce household poverty.

  28. Why is the mango called Cinderella species?

  A. It often appears in fairy tales.

  B. It is especially suitable for girls.

  C. Its importance fails to be recognized.

  D. It looks common but is very popular.

  29. What is stressed in Paragraph 2?

  A. The hard life of cocoa farmers.

  B. The great demand for chocolate.

  C. The reasons for chocolate’s popularity.

  D. The reasons for replacing cocoa with mangoes.

  30. What makes mangoes a replacement for cocoa?

  A. Similar output.

  B. Similar nutritional value.

  C. Similar growth conditions.

  D. Similar processing method.

  31. The underlined word “boon” in Paragraph 3 can best be replaced by_______ .

  A. disagreement B. gift

  C. challenge D. barrier

  D

  In the year 1963, animal experimenter Clarence Little and his team carried out an experiment on more than 100 rats. The group of scientists used products such as cigarettes to test for lung cancer, and they were trying to find a cure.

  They could not find any product to produce lung cancer or anything compared to the extremes that humans go through on the rats. Medical professionals are still insisting that they need to test on animals to cure cancer and other diseases. However, if rats can’t contract the same diseases, it means there is no real point in testing them.

  Since 1963, scientists have slowly been realizing that using animals in their experiments is not important. Medical advances have even been delayed due to experimenting on animals. Some data from U.S. Food and Drug Administration show that 92 percent of drugs which pass animal trials are found to be unsafe or ineffective in human trials and never reach the market.

  Animal experimentation is less efficient and reliable than many non-animal methods of experimentation. The most hated and known practice of animal experimentation is “pound seizure” which is where animal shelters sell animals to some labs and companies to test their products.

  It’s shocking that some countries still allow testing on animals. There are many products that have not been tested on animals and they work even better than the products that have been. So the need for animal experimentation is zero; there are so many ways to get around it. Stopping animal experimentation could be easily done in a few steps: reducing the number of animals being tested, reducing the types of animals that are tested, making sure the outcome will be actually used and needed, and finally replacing all animals with non-living models.

  32. Why did scientists do an experiment on rats in 1963?

  A. To look for a treatment for lung cancer.

  B. To improve the quality of some medicines.

  C. To find out how people suffered from cancer.

  D. To look for the cause of an unknown disease.

  33. What is a negative effect of using animals in experiments?

  A. Animals can easily become extinct.

  B. Medical advances are slowed down.

  C. A disease is easily spread among the animals.

  D. Medicines used on animals are not accepted by men.

  34. What do we know about “pound seizure”?

  A. It’s widely used.

  B. It’ s a non-animal test.

  C. It helps stop animal testing.

  D. It’s reliable but not efficient.

  35. Which of the following does the author agree with?

  A. Non-living models reduce the cost of experi¬ments. .

  B. The government is to blame for medical tests.

  C. It’s easy to find a better way to test medicines.

  D. There is no need to test medicines on animals.

  第二节(共5小题;每小题2分,满分10 分)

  根据短文内容,从短文后的选项中选出能填入空白处的最佳选项。选项中有两项为多余选项。(注意:将答案转涂到答题卡时,若选A, B, C,D则直接涂在机读卡上,若选E则同时涂A, B,若选F则同时涂B, C,若选G则同时涂C, D。)

  Anyone who keeps a garden knows that most plants bend towards the sun to catch as much sunlight as they can. This is called “solar tracking”.

  36 If you observe a sunflower in the early morning, you will find it turning its face eastward; at sunset, on the other hand, its face will turn towards the west.

  37 In other words, what happens if you take a sunflower out of the garden bed and keep it in a completely dark room?

  Surprisingly, the sunflower will continue its solar tracking without sunlight. It will bend, just as it did outside, eastward at sunrise and westward at sunset, even if there is not a single ray of light inside the dark room. This is a classic example of what scientists call a circadian rhythm (昼夜节律).What does it mean? 38 It’s not directed by the environ¬ment.

  Sunflowers do follow the sun, but they don’t just “seek” sunlight. 39 Although sunflowers don’t bend with any purpose in mind, some scientists think they have evolved their solar tracking system over millions of years. According to this theory, the evolution helps sunflowers catch light more effectively, and the ones that catch light best are most likely to survive. As a result of this natural selection, we now have sunflowers that bend automatically. 40 After a certain stage sunflowers stop following the sun and only face east.

  A. Sunflowers are a well-known example of this.

  B. Sunflowers are also an important source of food.

  C. Wild sunflowers seen on roadsides don t follow the sun.

  D. It is a daily cycle of behavior that is inside of living things.

  E. However, that only happens when sunflowers are young plants.

  F. But what if there’s no sun for the sunflower to guide its solar tracking?

  G. This is clear from the fact that sunflowers in darkness bend just as well.

  第三部分 英语知识运用(共两节,满分45分)

  第一节 完形填空 (共20小题;每小题1.5分,满分30分)

  阅读下面短文,从短文后各题所给的四个选项(A、B、C和D)中,选出可以填人空白处的最佳选项,并在答题卡上将该项涂黑。

  One Sunday, I drove my wife to work. On my 41 she asked me to do some shopping at the local shopping centre. Outside I noticed a(n) 42 guy. He sat on the floor with a small cup asking for 43 and a little dog at his side, keeping 44 .

  The shoppers walked past, 45 him on such a very cold day without any mercy. I 46 my pockets. £1.50 was all the change I had and I 47 it in his cup and said to him to treat himself. 48 I left him, I thought to myself: Is that really a treat? Leave this guy in the cold and 49 throw him £1.50! I knew this wasn’t 50 .

  I collected my shopping and as I made my way back, I thought what I could do to help this guy. So I went into a bakery and bought some 51 for him. But what about his 52 ? Again, I went to a shop, grabbing some doggy chocolate treats.

  Coming 53 ,I sat down, introduced myself and gave him the food. He was 54 and couldn’t believe it, but he kept shaking my hand and 55 me. Then I left. As I returned to my car, I 56 . The guy was eating his food and filling a bowl of water for his dog.

  Christmas is about 57 and helping others. 58 everything you have, for example, the rich food in your fridge over the Christmas period and your sofa. On the contrary, this guy had 59 of these. I’ve helped him for a day or two and hope that you may 60 a thought next time you see someone on the street.

  41. A. return B. Travel C. business D. celebration

  42. A. homeless B. Strong C. annoying D. intelligent

  43. A. treat B. Water C. money D.respect

  44. A. company B. Balance C. peace D. silence

  45. A. helping B. Watching C. pushing D. ignoring

  46. A. checked B. Filled C. picked D. shook

  47. A. collected B. Shot C. moved D. placed

  48. A. Since B. As C. If D. Unless

  49. A. even B. Yet C. just D. still

  50. A. right B. Common C. special D. clever

  51. A. bowls B. Pots C. material D. food

  52. A. cat B. Dog C. son D. daughter

  53. A. forward B. In C. out D. down

  54. A. honored B. Astonished C. frightened D. satisfied

  55. A. questioned B. Paid C. comforted D. thanked

  56. A. looked around B. looked out

  C. looked up D. looked back

  57. A. giving B. Suggesting C. accepting D. abandoning

  58. A. Store B. Consider C. Show D. Donate

  59. A. many B. All C. few D. most

  60. A. express B. Spare C. share D. Bear

  第II卷

  第二节 语法填空(共10小题;每小题1.5分,满分15分)

  阅读下面短文,在空白处填入适当的内容(不多于1个单词)或括号内单词的`正确形式。

  In many places today, children start primary school at around the age of six. However, because it is more likely now that both parents work, children have little opportunity 61 (stay) in their own home up to that age. Instead, they will probably go to a nursery school when they are much 62 (young).

  While some people think this may be damaging to children’s 63 (develop), or to children’s relationship with their parents, in fact there are many 64 (advantage) of experiencing school early.

  First, children will learn to interact with a lot of different people. They are 65 (general) more confident and independent than those who are not used to strangers or new situations. Such children may find their first day at school at age of six very 66 (frighten) and this may have a negative effect on how they learn.

  Another advantage of going to school at 67 early age is that children develop faster socially. They make friends and learn how to get on 68 other children of their age. This is not often possible at home because they are the only child 69 because their brothers or sisters are older or younger.

  So overall, I believe that 70 (attend) school from a young is good for most children. They still spend plenty of time at home with their parents, so they can benefit from both environments.

  第四部分:写作(共两节,满分35分)

  第一节 短文改错 (共10小题;每小题1分,满分10分)

  假定英语课上老师要求同桌之间交换修改作文,请你修改你同桌写的以下作文。文*有10处语言错误,每句中至多有两处。每处错误仅涉及一个单词的增加、删除或修改。

  增加:在缺词处加一个漏字符号(∧),并在其下面写出该加的词。

  删除:把多余的词用斜线(\)划掉。

  修改:在错的词下划一横线,并在该词下面写出修改后的词。

  注意:1.每处错误及其修改均仅限一词;

  2. 只允许修改10处,多者(从第11处起)不计分。

  When my parents told me that we will go to Wuhan for holiday by air, I really felt a little nervous. After all, it was my first time ever to take plane. We got on the plane after check in. Everything was so very strange to me that I didn’t dare to touch anything. But my nervousness didn’t last long due to the help of the hostesses. With their patient explanations, I soon got familiar to the environment. Thing went smoothly and I was enjoying the flight when suddenly we were warning to fasten the seat belt for the coming strong air flow. I became scared at the news, but they turned out to be a small incident during the whole journey. The plane soon became steadily and we landed safely at Wuhan Tianhe Airport one hour ago.

  第二节:书面表达(满分25分)

  假定你是李华,本学期你校开设了英语写作课,由来自美国的外教Shelly来授课。由于你在英语写作方面感到很困难,故打算写一封电子邮件向她求助,要点包括:

  1.告诉她你的困难;

  2.寻求帮助:如何提高写作技能,如何扩大词汇量,希望推荐写作书目。

  注意: 1. 词数100左右(开头已给出,但不计入总词数);

  2. 可以适当增加内容,使内容充实、行文连贯。

  Dear Shelly,

  I’m Li Hua, one of your students in the writing class.

  _


英语高考北京模拟试卷及答案(扩展4)

——届淄博市高考英语二模拟试卷及答案

届淄博市高考英语二模拟试卷及答案1

  第一部分 听力(共两节,满分30分)

  第一节(共5小题;每小题1.5分,满分7.5分)

  听下面5段对话。每段对话后有一个小题,从题中所给的A、B、C三个选项中选出最佳选项,并标在试卷的相应位置。听完每段对话后,你都有10秒钟的时间来回答有关小题和阅读下一小题。每段对话仅读一遍。

  1. Where was the woman’s phone yesterday?

  A. At her home. B. In a restaurant. C. In her office.

  2. Why didn’t the woman buy the coat?

  A. She disliked the colour. B. It didn’t fit her. C. The price was high.

  3. Why does the woman thank the man?

  A. He attended her party. B. He got her known to his friends. C. He found a new school for her.

  4. What’s the relationship between Lia and the woman?

  A. Neighbors. B. Classmates. C. Sisters.

  5. Which necklace will the woman buy?

  A. The red one. B. The blue one. C. The clear one.

  第二节(共15小题;每小题1.5分,满分22.5分)

  听下面5段对话或独白。每段对话或独白后有几个小题,从题中所给的A、B、C三个选项中选出最佳选项,并标在试卷的"相应位置。听每段对话或独白前,你将有时间阅读各个小题,每小题5秒钟;听完后,各小题将给出5秒钟的作答时间。每段对话或独白读两遍。

  听第6段材料,回答第6、7题。

  6. What are the speakers mainly talking about?

  A. How to get to the Train Station. B. How to go to the bus stop. C. How to save money.

  7. Which bus should the woman take first?

  A. Bus No. 6. B. Bus No. 7. C. Bus No. 8.

  听第7段材料,回答第8、9题。

  8. How much did the man pay for his TV?

  A. $700. B. $800 C. $1,500

  9. What does the woman think about the man?

  A. Interesting. B. Unrealistic. C. Silly.

  听第8段材料,回答第10至12题。

  10. What’s Tom’s summer vacation plan?

  A. Do some part time job. B. Have a concert. C. Practice his guitar.

  11. Why does Cindy look for a job?

  A. To get some experience. B. To pay for her school. C. To save money to travel.

  12.What will Cindy do this afternoon?

  A. Go to the community center. B. Send emails. C. Buy concert tickets.

  听第9段材料,回答第13至16题。

  13. What time is it now?

  A. 8:00 a.m. B. 9:00a.m. C. 6:00 p.m.

  14. What was the man doing when the woman called him?

  A. He was in a shower. B. He was working. C. He was sending a package.

  15. Where will the speakers meet after work?

  A. At the man’s company. B. At the theater. C. At the post office.

  16. Who will give a dancing show?

  A. The woman s brother. B. The woman’s sister. C. The woman.

  听第10段材料,回答第17至20题。

  17. How many gold medals did Simone win in 2016 Olympics?

  A. 3. B. 4. C. 5.

  18. Who is Aimee Boorman?

  A. Simone’s mother. B. Simone’s teammate. C. Simone’s coach.

  19. When did Simone first take part in the World Championships?

  A. 2004. B. 2013. C. 2016.

  20. What do people think about Simone’s personality?

  A. She is optimistic. B. She is shy. C. She is tough.

  第二部分:阅读理解(共两节,满分40分)

  第一节(共15小题;每小题2分,满分30分)

  阅读下列短文,从每题所给的四个选项(A、B、C和D)中,选出最佳选项。

  A

  The United Kingdom has voted to leave the European Union, a historic decision that will reshape the nation^ place in the world and is already upsetting governments across the continent. The surprising turn of events caused a fall in the financial markets, with the value of the British pound and stock prices falling.

  The campaign to leave the EU won by 52 percent to 48 percent. More than 17 million people in the UK—made up of England, Scotland, Wales, and Northern Ireland — voted to sever ties with the EU, a Political and economic partnership comprising 28 countries across Europe. About 16.1 million voted to remain.

  The result raises serious questions about the future of the EU. The British go out from the union-nicknamed Brexit — is just the latest in a series of crises to annoy the EU. The alliance is already dealing with economic bad results from the global financial collapse of 2008, along with a record arrival refugees to Europe due in part to Syria’s civil war.

  Some British citizens believe the EU has too much influence over Britain’s authority and that the country’s industries would be more profitable on their own. Others want Britain to reclaim control of its borders and reduce how many people enter the country to work They say the EU’s open-border policy allows unlimited numbers of immigrants to come in from poorer nations—and that those people take jobs from British workers. Those who campaigned to stay in the EU, meanwhile, argued that the UK and its economy is stronger within the alliance.

  Now the UK will become the first nation to leave the European Union. The decision is a huge blow the alliance because Britain is one of its most powerful members. “The main impact will be massive disorder in the EU system for the next two years,” said Thierry de Montbrial, of the French Institute of International Relations. “There will be huge political change costs on how to solve the British exit.” The decision could also have a domino effect, he warned, with other countries following the UK’s lead and exiting the EU.

  21. What does the underlined word “sever” mean?

  A. Settle down. B. Take over. C. Break off. D. Build up.

  22. Most British wanted to leave the EU because they believe .

  A. they can help immigrants to create more job chances

  B. they expect the EU to have an open border policy

  C. they want to set up their own authority

  D. they will lead a better life on their own

  23. What did Brexit make Thierry worry about?

  A. More countries will do as the UK does. B. Scotland win their independence from the EU.

  C. The EU will pay back more money to the UK. D. The economy in the EU will suffer loss.

  24. What might be the most suitable title for the passage?

  A. The UK Wins Independence with Votes B. The UK Votes to Leave the European Union

  C. The UK Makes a Historic Decision Again D. The UK Remains Separated from the World

  B

  Scientists announced on Thursday, February 11, 2016, at a press conference in Washington, D. C that the more than half-century search for gravitational waves finally succeeded. They confirmed that they had detected gravitational waves caused by two black holes mixing together about 1. 3 billion years ago. It had been the first time the phenomenon was directly detected since it was predicted by Albert Einstein l00 years ago.

  A China led space gravitational wave detection project is recruiting (招募) researchers from around the world, china news, com reported. The Tianqin project was started by Sun Yat-sen University in Zhuhhai. It centers on research of key technologies in space gravitational wave detection. To make the project more competitive,global talents in areas such as gravity theory,space gravity experiments and precision measurement are sought with an annual salary of up to 1 million yuan ($ 153.000). according to the report.

  Gravitational waves are not easily detectable. When they reach the Earth, an extremely sensitive detector is needed, as other sources of noise can overwhelm the signal. Luo Jun. president of Sun Yat sen University said the detection of gravitational waves is a huge challenge and the project reflects the research competence of China.

  The 15-billion yuan project aims to send wave-detection satellites into space in 15 to 20 years with construction of a research center and observatory to be started soon. Researchers will likely cooperate with other scientists from the European Space Agency’s LISA Project.

  Gravitational waves are ripples (涟漪) in space time generated during the earliest moment of the universe, when the universe experienced a brief but disordered period of expansion. The detection of gravitational waves is believed to help scientists understand more mysteries of the universe.

  25. What can we learn about gravitational waves?

  A. They caused black holes to mix together. B. They came out about half a century ago.

  C. They wore proved true by Albert Einstein. D. They have been discovered recently.

  26. According to the passage, the Tianqin project is intended to .

  A. develop technologies to detect space gravitational waves

  B. attract talented researchers from across the globe

  C. conduct various space gravity experiments

  D. seek precision measurement in space

  27. The underlined word “overwhelm” in the third paragraph probably means “ .”

  A. absorb B. replace C. remove D. cover

  28. The main purpose of the passage is to .

  A. explain the importance of the gravitational wave detection

  B. present some new findings in space exploration

  C. introduce a gravitational wave research project

  D. encourage international cooperation in scientific research

  C

  Those accustomed to browsing through thousands of books in large bookstores may find Japan’s Morioka Shoten a little strange. That’s because this tiny bookstore that is located in Ginza,Tokyo sells only a single book at a time.

  Opened in May 2015, Morioka Shoten is the brainchild of Yoshiyuki Morioka. He began his career as a bookstore clerk in Tokyo’s Kanda district before branching out to open his own store. It was here while organizing book reading and signing that he realized that customers usually came into the store with one title in mind. Morioka began to wonder if a store could exist by selling multiple copies of just one single book. In November 2014, he partnered with Masamichi Toyama to establish a unique bookstore with the philosophy of “A Single Room with a Single Book”.

  The selections that are picked by Morioka change weekly and vary widely to attract customers with different interests. Recent choices include The True Deceiver, an award-winning Swedish novel by Tove Jansson, Hans Andersen’s Fairy Tales, and a collection of Karl Blossfeldt’s photography of plants. Morioka has also selected books written by a famous Japanese author Mimei Ogawa.

  To highlight his only offering, Morioka often uses clever tricks. For example, when selling a book about flowers, the storekeeper decorated his shop with the ones that had been mentioned in the book. He also encourages authors to hold talks and discussions so they can connect with customers. Morioka says his goal is for the customers to experience being inside a book, not just a bookstore.

  Risky as the idea might seem, things appear to be going well. The storekeeper says he has sold over 2,100 books. Things can get better given that his bookstore is becoming increasingly popular not just among the locals but also visitors form other countries.

  29. Why is Morioka Shoten unique?

  A. It is popular with feigners. B. It sells books of different topics.

  C. It is decorated with colorful flowers. D. It sells various copies of a book in a week.

  30. Why does Morioka encourage authors to hold talks?

  A. To introduce his bookstore. B. To advocate his philosophy.

  C. To make books better understood. D. To help readers make more friends.

  31. What’s the author’s opinion about the bookstore’s future?

  A. Risky B. Promising. C. Unpredictable. D. Hopeless

  D

  After a friend recommended that he join a secret Google project six years ago, Brian Torcellini became a driver in a driverless car.

  Torcellini,31, leads a crew of test drivers who are legally required to ride in Google’s fleet of 48 driverless cars . They only take a control in emergencies. Otherwise, they make observations that help the company’s engineers improve the driverless cars.

  The driverless cars already have covered more than 2 million miles in six years of testing on private tracks, on highways and city streets. The vehicles have traveled more than half that distance in automated mode (自动模式),with one test driver in place to take control of the car if the technology fails or a potentially dangerous situation arises. Meanwhile, another driver sits in the front passenger seat taking notes of the problems that need to be fixed.

  The job of the test driver requires a sense of adventure, something Torcellini acquired when he began to surf in high school. His other hobbies include fishing and scuba diving.

  While the engineers who are programming the robot cars have technical backgrounds, most of the test drivers don’t. Torcellini worked in a book store. He dreamed of becoming a writer. He ended up at Google in 2009. Espinosa, 27, was working in a bicycle shop before he was hired as a test driver two years ago . Stephanie Villegas, 28, was a swim instructor, before becoming a test driver. Other test drivers were former soldiers and former photographers. They all share at least one thing in common: spotless driving records. Besides this, the job also requires a combination of good judgment, patience and fearlessness.

  Before they are entrusted (托付) with the cars,Google’s test drivers must complete three-week training courses. The drivers are taught to take control of the robot car whenever there is any moment of doubt or danger.

  Google employs “dozens” of test drivers but won"t reveal the precise number . California law requires two test drivers per vehicle.

  32. What do you know about Brian Torcellini ?

  A. He is in charge of a crew of test drivers. B. He used to be a swim instructor.

  C. He led a secret Google project. D. He has technical backgrounds.

  33. The driver sitting in the front passenger seat is responsible for .

  A. fixing the car problems B. driving in emergencies

  C. recording the car problems D. monitoring the car driver

  34. We can infer that .

  A. Stephanie Villegas is interested in scuba diving B. Brian Torecellini is very fond of literature

  C. Espinosa is an excellent cyclist in California D. driverless cars are very popular in California

  35. The underlined word “this” refers to .

  A. having good judgments B. having clean driving records

  C. breaking the traffic rules D. having technical backgrounds

  E

  根据短文内容,从短文后的选项中选出能填入空白处的最佳选项。选项中有两项为多余选项。

  Tips for Teaching Kids Responsibility

  Your best friend has a teenager who helps with housework without being asked. Your cousin has a one-year-old child that puts her bottle in the sink when she finishes the milk, but no reminders are needed. 36 Raising helpful, good kids who know how to make a sandwich is not a fantasy! The following tips can give you a better chance of raising a responsible child who then grows into a responsible adult.

  37 You can’t suddenly spring responsibility on a teenager and expect he will know how to follow through. He needs time to practice and totally understand what responsibility means.

  Let kids help you. Don t complain when it"s time to do housework. Smile and invite your son to help. 38 He will take these good feelings and learn to take ownership of his home and feel pride in keeping it up.

  39 Make responsibilities age appropriate. You can show how to complete small tasks in daily life and perform them at a child’s skill level. For example, if your kid wants a snack, show him where the apples are and how to wash one off.

  Teach your child consequences. Learning to take care of his things also helps a child develop a sense of responsibility for his actions. Get your son to clean up after an art project, and inform him that he won^ be able to play with his crayons the next day if he leaves a messy table. 40 The more you carry out the rules, the more likely he is to clean up without being asked.

  A. Have a sense of responsibility.

  B. Why do they make a sandwich for kids?

  C. Show kids how to perform responsibility.

  D. Where do these wonderful children come from?

  E. Start to teach responsibility to kids early.

  F. When your child is invited to participate, he feels valued.

  G. Then take away his supplies if he does not take his responsibility.

  第三部分:英语知识运用(共两节,满分45分)

  第一节 完形填空(共20小题;每小题1.5分,满分30分)

  阅读下面短文,掌握其大意,然后从36-55各题所给的A、B、C、D四个选项中,选出最佳选项,并写在答题纸上。

  At an airport I overheard a father and a daughter in their last moments together. He said to his daughter, “I love you. I wish you 41 She said, “Daddy, our life together has been more than enough. Your love is all I ever 42.I wish you enough, too They kissed goodbye and she left.

  The father stood there 43 with tears in his eyes. I tried not to interrupt his privacy, but he 44 me and greeted me by asking, “Did you ever say goodbye to someone knowing it would be 45?” “Yes, I have,” I replied.

  Saying that brought back my memories of expressing my love and 46 of all my dad had done for me. Recognising that his days were 47, I spent the time telling him face to face how much he 48 to me. So I knew what this man was 49.

  “Forgive me for asking, but why is this a goodbye forever?” I asked. “I am old and she lives much too faraway I have 50 ahead and the reality is that her next trip back will be 51 for my funeral,” he said. “When you were saying goodbye, I heard you say ‘I wish you enough’. May I ask what that means?” He began to 52. “That’s wish that has been handed down from other 53.” He paused for a moment looking up as if trying to remember it in detail, and he smiled even more .

  “When we said ‘I wish you enough’, we 54 the other persons to have a life filled with enough good things 55 them,” he continued the following as if he were 56 it from memory. “I wish you enough sunlight to keep your attitude 51. I wish you enough rain to appreciate the sun more. I wish you enough happiness to keep your spirit alive. I wish you enough pain so that the smallest joys in life appear 58 bigger I wish you enough gain to satisfy your needs. I wish you enough ‘loss’ to appreciate all that you 59. I wish you enough ‘Hellos’ to get you 60 the final ‘goodbye’.” he then began to sob and walked away.

  41. A. enough B. luck C. success D. health

  42. A. support B. praise C. judge D. need

  43. A. carefully B. silently C. seriously D. gratefully

  44. A. noticed B. monitored C. thanked D. observed

  45. A. endless B. meaningful C. forever D. temporarily

  46. A. appreciation B. admission C. enthusiasm D. sacrifice

  47. A. dark B. short C. wonderful D. Bitter

  48. A. explained B. lent C. meant D. introduced

  49. A. wondering B. experiencing C. approving D. imagining

  50. A. differences B. passions C. wonders D. challenges

  51. A. probably B. extremely C. gradually D. clearly

  52. A. smile B. shout C. suffer D. Cry

  53. A. countries B. generations C. mothers D. communities

  54. A. admitted B. put C. thought D. wanted

  55. A. promising B. teaching C. surrounding D. charging

  56. A. reminding B. losing C. forgetting D. reciting

  57. A. serious B. opposite C. bright D. open

  58. A. only B. much C. any D. just

  59. A. treat B. decide C. donate D. possess

  60. A. through B. in C. along D. across

  第II卷(非选择题,共50分)

  第二节 语法填空(共10小题,每小题1.5分,满分15分)

  阅读下面材料,在空白处填入适当的内容(不多于3个单词)或括号内单词的正确形式 小题;每小题1.5分,满分15分)

  Until now, many people 61 (see) Minions. No, 62 I am talking about is not the recent movies, but the village in England with the same name. Now it may be 63 (difficult) to find it than before, as the village has had to take down its sign because it worries about 64 (safe). The village put up a special sign in May as pan of a deal with Universal Studio 65 (promote) the movie Minions. The sign 66 (feature) three of the cute characters posing next to the village’s name. And the Minions sign proved popular with travelers, as 67 (apparent) shown by the many pictures on special media posted by drivers 68 pulled in to take Photos with the signs. Some local businesses wanted to keep the sign place to help with tourism, but Carl Hearn, a local official, said officials had to take it down as they were worried about drivers 70 (stop) in front the sign in order to take photographs.

  第四部分 写作(共两节,满分35分)

  第一节 短文改错(共10小题;每小题1分,满分10分)

  假定英语课上老师要求同桌之间交换修改作文,请你修改你同桌写的以下作文。文*有10处语言错误,每句中最多有两处。每处错误仅涉及二个单词的增加、删除或修改。

  增加:在缺词处加一个漏字符号(∧),并在其下面写出该加的词。

  删除:把多余的词用斜线(\)划掉。

  修改:在错的词下划一横线,并在该词下面写出修改后的词。

  注意:1.每处错误及其修改均仅限一词;

  2.只允许修改10处,多者(从第11处起)不计分。

  Insurance companies are normally willing to insure anything, but it must be unique to insure a dish. It was a unusual pie dish, for it was eighteen feet in long and six feet in width. They had been purchased by a local authority so that an enormous pie could baked for an annual fair. The pie committee decided that the best way to transport would be by a c*, so they insured it for the trip. Short after it was launched, the pie committee went to the local inn to celebrate. In the same time, a number of teenager climbed on to the dish and held a little party of their own. Dancing proved to be more than the dish could bear, but during the party it capsized (倾覆) and sink in seven feet of water.

  第二节:书面表达(满分25分)

  某英语杂志正在举办“分享我的格言”的活动,你最喜欢的英语格言是Robert Collier的“Success is the sum of small efforts, repealed day in and day out”。请根据下面的写作提示写一篇投稿,内容包括:

  1.你如何理解这句格言;

  2 .举例说明你的观点。

  注意:词数100左右。

  Dear Editor,

  I"m Li Hua, a Senior 3 student. I feel it a great honor to get involved in such a meaningful activity.

  Yours sincerely,

  Li Hua


英语高考北京模拟试卷及答案(扩展5)

——届佛山市高考英语模拟试卷及答案

届佛山市高考英语模拟试卷及答案1

  第Ⅰ卷

  第二部分 阅读理解(共两节,满分40分)

  第一节 (共15小题;每小题2分,满分30分)

  阅读下面短文,从每题所给的ABCD四个选项中,选出最佳答案,并在答题卡上将该项涂黑。

  A

  Mobile phones and iPads are now wonderful tools for learning and teaching English. With the help of the thousands of education and language apps available online, students can make effective use of them to learn in an interactive environment. Given below is a list of some of the best apps for learning and teaching English.

  Grammar Up

  Grammar is the heart of any language and, thus, the Grammar Up app is one of the most highly recommended apps for English learners. This app is specifically designed for people who want to improve their grammar, vocabulary and word selection. It consists of more than 1,800 multiple-choice questions grouped under twenty different grammar categories.

  iFPoems

  iFPoems is a fantastic app for poetry lovers. This app consists of a great collection of poems from famous poets around the world. It also allows users to save their favorites along with recording their own readings of the poems.

  iBooks

  iBooks is a complete book library for the iPad users. It includes the iBookstore and allows users to download and read books. The app features a beautiful bookshelf, on which you can browse various books or even add notes to your favorite paragraphs in the book. You can also create your own texts and upload them into the app.

  SpeakingPal

  Recommended as one of the best apps for English educators and learners, SpeakingPal allows you to record your speech and compare it with a native English speaker. This amazing app combines video with automatic speech recognition software that tests your English speaking skills. You just have to speak with a video character and get immediate response on how well you speak English.

  21. What do these apps have in common?

  A. They can be found online. B. They are free of charge.

  C. They improve users’ grammar. D. They’re designed by teachers.

  22. What can we do with iBooks?

  A. Download books for free. B. Write notes on it with pens.

  C. Save notes and texts. D. Create a bookstore.

  23. What is special about SpeakingPal?

  A. It offers video call service. B. It evaluates users’ speech.

  C. It offers talks with a native speaker. D. It includes English speaking contests.

  B

  I don’t want kids. I know I wouldn’t make for a very good mother. It’s not that I’d drop them or forget them at the grocery store or anything like that but they’re not a good fit for me and the life I’ve created for myself. I’d rather be in Cambodia kissing elephants or cage diving with sharks in Cape Town, or wandering the streets of Paris, a city that has become my second home, than trapped at home doing the mom thing.

  The itch to become a mother isn’t in me but the itch to travel, and travel far and often, is.

  If the urge isn’t there, I don’t think it’s fair to live a life I don’t want and have a baby because someone else thinks I should. That would be awful.

  I like children. To say I love them wouldn’t be completely accurate. It’s not because I’m a monster — the same monster people who feel women should have children will call me — but I sim* don’t love them because I’m not around them much. Very few of my friends have children, and the ones who do I don’t see much because, well, they have children.

  I have two nephews in Colorado whom I love to pieces, but I don’t see them very often. I don’t go out there as often as I probably should because I’d rather be traveling. Some people may call this selfish, but I call it “living my best life.”

  When I talk to my sister about my nephews, everything they do and say is exciting to her. The joy my sister feels by just playing with them in the yard is the same joy I feel every time I get off a plane in a new country. I’d rather be on the beaches of Spain every summer while she prefers to spend her days at home with them, climbing trees and such. I’m fulfilled one way, while she’s fulfilled another way.

  I’m very much certain that at this point in my life my greatest fulfillment comes in the form of traveling and being free of kids. I can go anywhere at anytime; I don’t have to put my life on hold for someone else. It’s complete and total freedom.

  24. What does the underlined word “itch” in Paragraph Two mean?

  A. tradition B. competence C. plan D. desire

  25. Why is the author regarded as a monster?

  A. Because she doesn’t like kids. B. Because she doesn’t want children.

  C. Because she’s different from her friends. D. Because she rarely stays with children.

  26. Which is “living my best life” according to the author?

  A. To stay with her family. B. To see her nephews often.

  C. To visit a new country. D. To talk with her sister.

  27. What is the author’s purpose of writing the passage?

  A. To explain her choice of life. B. To persuade people to travel.

  C. To share her happiness in life. D. To introduce a unique lifestyle.

  C

  Long school holidays help students clear their heads, relax a little and stretch their brains in different ways.

  Traditionally, schooling schedules have long been based more on the needs of society than on efficient education strategies. For instance, in Iceland it was traditionally arranged to allow children to help out with harvest. Other countries have long summer vacations because it is sim* too hot to study. In other areas where religion plays an important role in the holiday season, the religion calendar then calls for the break, so that students can have ample time to participate in the religious ceremonies and traditions of seasonal holidays.

  There are some obvious advantages to a vacation that lasts longer. Families who live a long distance from relatives are able to travel together, sometimes across the ocean, in order to connect with their loved ones. Even if there is no visiting family far away, a long holiday is a great opportunity for kids to interact with the community. From an educational point of view, a child that is able to experience another culture can gain valuable language, history, and social skills.

  Long school holidays are also needed in schools that have become overly academic and too exam-focused in recent years, which requires a considerably longer holiday break from such a “marathon pumping (灌输) of knowledge,” and it makes great sense!

  Great psychologists, like Freud and Piaget, have stressed play’s central role in the formation of fully-rounded (全面发展的`) individuals, so the long holiday achieves this by allowing students to explore the world outside school. The students get to participate in activities that would be difficult to do during a regular school term.

  As a consequence, long school holiday can be quite constructive to our students!

  28. What traditionally determines the arrangement of school holidays?

  A. Local needs. B. Cultural activities.

  C. Religious customs. D. Academic performance.

  29. Which is one of the advantages of long holidays?

  A. Students run away from exams. B. Children can learn science and history.

  C. Fewer courses are required at school. D. Family bonds are strengthened.

  30. Freud and Piaget may probably agree that ________________.

  A. schools need longer holidays B. holiday activities are necessary

  C. play is more important than learning D. students should travel around the world

  31. What is the best title of the passage?

  A. Traditional School Holidays B. Activities for Long Holidays

  C. Benefits of Long Holidays D. Advice from Psychologists

  D

  Every February 14, people worldwide demonstrate their affection for family members, friends, teachers, and significant others with a gift or two. However, this display of love does not come cheap. 55% of Americans celebrate Valentine’s Day in 2017 by spending an average of 136.57 dollars each. It is no wonder that retailers (零售商) love the holiday.

  In 1861, Richard Cadbury, a man from a British chocolate company came up with the idea of packaging chocolates in heart-shaped boxes for Valentine’s Day. Not surprisingly, they were an instant success, and a tradition was born. Today, Americans spend over one billion dollars handing out boxes of chocolates to their loved ones. Sweethearts, the heart-shaped candy, are also very popular. According to the manufacturer, a majority of the 8 billion pieces of candy made annually are sold between January 1 and February 14.

  Massachusetts businesswoman Esther Howland popularized Valentine’s Day greeting cards in America. Often called “The Mother of the American Valentine”, the artist was the first to sell handmade paper cards with messages of love to retailers. With over 145 million cards bought in the US and over a billion worldwide, Valentine’s Day is now one of the biggest business chances for greeting card companies. It is no wonder that the day is often referred to as a “Hallmark Holiday”.

  Flowers are also very popular. About 51% of people, mainly men, buy red roses for their loved ones. Given that Valentine’s Day is very popular for wedding proposals (求婚), it should not come as a surprise that a lot of the money spent on this day is used to buy fine jewelry. With a quarter of Americans treating their loved ones to a romantic meal, the day is also much welcomed by restaurant owners.

  Pope Gelasius, the man who made February 14 Valentine’s Day in 496 AD, would have never imagined that this festival would change into such an expensive celebration!

  32. What does the first paragraph mainly tell us?

  A. People buy their friends gifts on February 14.

  B. Valentine’s Day is celebrated every year.

  C. Valentine’s Day is a good time to show love.

  D. A lot of money is spent on Valentine’s Day.

  33. Which is true about Richard Cadbury’s idea?

  A. It resulted in a new taste of chocolate.

  B. It led to the birth of a new tradition.

  C. It was put forward on Valentine’s Day.

  D. It helped the company earn one billion dollars.

  34. Valentine’s Day is referred to “Hallmark Day” because _______________.

  A. people get greeting cards on that day

  B. it was popularized by Miss Howland

  C. it offers cards companies big business chances

  D. retails help send messages of love on that day

  35. What is probably a necessary gift for a wedding proposal according to the passage?

  A. Heart-shaped candy. B. Messages of love.

  C. A romantic meal. D. A piece of jewelry.

  第二节 (共5小题,每小题2分,满分10分)

  根据短文内容,从短文后的选项中选出能填入空白处的最佳选项。选项中有两项为

  多余选项。

  Information Overload

  In modern society, if we’re trying to make a decision, we often have so much information that we get confused, and we don’t know what to do. This state is known as information overload. 36 It can cause stress, frustration and reduced productivity. But what can we do in the face of information overload?

  • Plan for only one time each day to check e-mail, social messaging sites, chat rooms, etc. Don’t allow yourself to check multiple times, unless you truly are waiting for an important e-mail. 37 And that eats up your valuable time before getting away.

  • If you are suffering from too many electronic interruptions during the weekday, ask people to call or text you during work hours only if it’s really an emergency. 38

  • Remind yourself that it’s okay to not know everything. In fact, it’s impossible to keep up with the pace of the information superhighway. The sooner you accept that, the happier you’ll be. 39 It is good to fall behind on the information that really isn’t worth your attention.

  • 40 Set aside a regular time each week where you and other family members do not use any kind of electronic media technologies, including television. It could be something you do every weekend, or perhaps an hour or two every evening.

  A. Just catch up with the happiest moment.

  B. Know what’s worth knowing and what isn’t.

  C. Almost everyone suffers from it to some degree.

  D. Spend time with your family free from electronic products.

  E. We need to find some effective ways to process the information.

  F. Each time you go online, you run the risk of being addicted to it.

  G. Otherwise they and you end up stealing time from your employer.

  第三部分 英语知识运用 (共两节,满分45分)

  第一节 完形填空(共20小题;每小题1.5分,满分30 分)

  阅读下面短文,从每题所给的A、B、C 和 D 四个选项中,选出最佳选项,并在答题卡上将该项涂黑。

  Most 7-year-old girls are playing Barbie and jumping rope. Though Sailor Gutzler may have enjoyed those 41 , she was also an active participant in 42 activities with her father. He was a pilot and he taught her 43 skills. She actually used these 44 when her family’s twin-engine plane 45 upside down in rural Kentucky. They’d been traveling 46 to Illinois in cold weather from a Christmas vacation in Key West, Florida.

  Sailor had broken bones, but when she couldn’t 47 her father, mother, sister or cousin after the crash, she 48 herself from the remains, lit a stick on the burning wing of the plane and walked 49 for a mile through the cold darkness of the wooded Kentucky 50 .

  Sailor 51 the nearest house, hoping to get help for her 52 . She told the woman who answered the 53 that she’d been in a(n) 54 and that her parents were dead.

  Though the temperature was 5℃ below zero, Sailor was wearing clothing meant for the

  55 Florida and only had on one sock. 56 , she’s a survivor.

  Though her father had been 57 since he was 16 and was a flight 58 for more than 4,000 hours, disaster still struck. He had made the same exact 59 many times but still, the weather won. 60 , he’d taught his daughter what to do and she did it.

  41. A. ropes B. sports C. activities D. dolls

  42. A. outdoor B. social C. school D. daily

  43. A. repair B. flight C. communication D. survival

  44. A. skills B. tricks C. thoughts D. activities

  45. A. flew B. landed C. swung D. appeared

  46. A. around B. home C. abroad D. everywhere

  47. A. wake up B. struggle for C. call out to D. get contact with

  48. A. hid B. supported C. pulled D. protected

  49. A. fast B. steadily C. heavily D. endlessly

  50. A. countryside B. tunnel C. wonderland D. desert

  51. A. got ready for B. went on for C. made it to D. looked forward to

  52. A. fear B. loss C. plane D. family

  53. A. question B. request C. girl D. door

  54. A. crash B. attack C. holiday D. dilemma

  55. A. remote B. warm C. rainy D. famous

  56. A. Actually B. Besides C. Finally D. Still

  57. A. practicing B. flying C. studying D. fighting

  58. A. agent B. planner C. steward D. instructor

  59. A. decision B. trip C. mistake D. plan

  60. A. Previously B. Accidentally C. Fortunately D. Amazingly

  第 II 卷

  第三部分 英语知识运用(共两节 满分45)

  第二节 (共10小题;每小题1.5 分,满15分)

  阅读下面短文,在空白处填入 1个适当的内容或括号内单词的正确形式。

  A balanced diet means the nutrition of three meals in a day should be balanced, and breakfast is very important to achieve that goal. Skipping breakfast 61 (cause) damage to health, apart from reducing work performance. 62 (study) have showed that if a person does not eat breakfast 63 (regular), he or she will be 64 a higher risk of various diseases.

  However, many people do not eat breakfast. A recent survey shows that 8.6 percent of people do not eat breakfast every day. The situation is even 65 (bad) in poverty-stricken areas.

  While most people are used to 66 (eat) breakfast, the quality of their meal is in question. About 25 percent of people do not eat a homemade breakfast, 67 more than 80 percent of people do not consume a healthy breakfast.

  Children, women, and people living in poverty-stricken areas are most likely to be affected 68 it comes to eating unhealthy breakfasts. However, old people 69

  (age) above 60 tend to have healthy breakfast, because this group, mostly retired, have time and 70 (patient) to cook breakfast themselves.

  第四部分 写作(共两节,满分 35)

  第一节 短文改错(共 10小题;每小题1分,满分10 分)

  假定英语课上老师要求同桌之间交换修改作文,请你修改你同桌写的以下作文。文*有10处语言错误,每句中最多有两处。每处错误仅涉及一个单词的增加、删除或修

  改。

  Ms. Trump is my history teacher. She loves a subject that she teaches and has a completely knowledge of it, too. History comes alive in his class, where no one talks about something but the topic. She likes to teach by telling stories, what always makes her lessons so interesting but easy to understand. Ms. Trump also has a good knowledge of other subject and often links it to what she teaches about. The best part about her is that she’s always nice to those students who needs more time to grasp the subject, encouraging them ask questions so that they can understand the subject better.

  第二节 书面表达(满分 25分)

  假定你是李华,是你校“读书俱乐部”的负责人,计划在下个月组织“英语读书节”。

  请给你的美国笔友Tony写信,介绍本次活动,并请他为本次读书节推荐阅读书目。

  注意:

  1. 词数100左右;

  2. 可以适当增加细节,以使行文连贯;

  3. 信的开头和结尾已给出,不计入总词数。

  Dear Tony,

  ...........................................................................................................................................................................................................................

  Looking forward to your early re*!

  Yours,

  Li Hua


英语高考北京模拟试卷及答案(扩展6)

——届宜春市高考英语模拟试卷及答案

届宜春市高考英语模拟试卷及答案1

  第一部分:听力(共两节,满分30分)

  做题时,先将答案标在试卷上,录音内容结束后,你将有两分钟的时间将试卷上的"答案转涂到答题卡上。

  第一节(共5小题,每小题1.5分,满分7.5分)

  听下面5段对话,每段对话后有一个小题,从题中所给的A、B、C三个选项中选出最佳选项,并标在试卷的相应位置。听完每段对话后,你都有10秒钟的时间来回答有关小题和阅读下一小题。每段对话仅读一遍。

  1. How will the girl’s mother pay for the CD?

  A. In cash. B. By cheque. C. By credit card.

  2. What will the speakers do in the afternoon?

  A. Build a tree house. B. Go mountain biking. C. Play beach volleyball.

  3. What does the woman ask the man to do?

  A. Drive a car. B. Move some boxes. C. Make a phone call.

  4. What are the speakers discussing?

  A. When to watch TV. B. Whether to watch a film. C. What program to watch.

  5. Why is the woman disappointed about the restaurant?

  A. The price is unacceptable. B. The waiter is unfriendly. C. The service is slow.

  第二节(共15小题:每小题1.5分,满分22.5分)

  听下面5段对话或独白。每段对话或独白后有几个小题,从题中所给的A、B、C三个选项中选出最佳选项,并标出试卷的相应位置。听每段对话或独白前,你将有时间阅读各个小题,每小题5秒钟;听完后,各小题将给出5秒钟的作答时间。每段对话或独白读两遍。

  听第6段材料,回答第6、7题。

  6. What are the speakers mainly talking about?

  A. Weekend plans. B. A city tour. C. Japanese food.

  7. Where will the speakers meet on Saturday night?

  A. At a concert hall B. At the woman"s home C. At a Japanese restaurant

  听第7段材料,回答第8、9题。

  8. Where does the conversation probably lake place?

  A. At a railway station. B. At an airport. C. At a bus stop.

  9. How is the weather now?

  A. Rainy. B. Snowy. C. Foggy.

  听第8段材料,回答第10至12题。

  10. Who is the Best Car Design Prize winner?

  A. Mr. Zarba. B. Mr. Giddings. C. Ms Miller.

  11. What do we know about the Best Car Design competition?

  A. The dinner will receive much money. B. There were few compe*s.

  C. It was very competitive.

  12. Where is the Hyatt Hotel?

  A. On the left side of Granville Avenue. B. Opposite a department store.

  C. Across from a bank.

  听第9段材料,回答第13至16题。

  13. Who is at Music Club?

  A. Katrina. B. Angelina. C. Sonya.

  14. What is Lucy doing at her club this month?

  A. Playing an instrument. B. Discussing short stories. C. Doing a science project

  15. What does Molly think of Film Club?

  A. Exciting. B. Interesting. C. Boring.

  16. Which club does Ella most probably go to?

  A. Photography Club. B. Book Club. C. Dance Club.

  听第10段材料,回答第17至20题。

  17. When should the listeners gather outside the school gale?

  A. At 7:45. B. At 8:00. C. At 8:55.

  18. What do the listeners need to do before Thursday?

  A. Invite their parents to the trip. B. Learn to make pizza. C. Give the speaker some money.

  19. What should the listeners take?

  A. Sandwiches. B. A pencil. C. A notebook.

  20. How does the speaker suggest parents ask questions?

  A. On the phone. B. By e-mail. C. Face to face.

  第二部分:阅读理解(共两节,满分40分)

  第一节(共15小题,每小题2分,满分30分)

  阅读下列短文,从每题所给的A、B、C和D四个选项中,选出最佳选项,并在答题卡上将该项涂黑。

  A

  Here are some great beach spots for your family to have some fun together.

  High Bar Harbor, New Jersey

  It sits on Long Beach Island, one of the most popular Jersey shores. But watch the water when the tide comes in, you may not be able to leave until the tide goes.

  How to Get There: Take Long Beach Blvd, north into the town of Barnegat Light and turn left on 20th St., a.k.a. Auburn Road, Go a 1/3 of a mile, and then bear right at the Y-shaped intersection (交叉点)onto Sunset Blvd. Take it to the end. You’ll see a footpath to the beach.

  Caladesi Beach, Florida

  To be honest, you"ll hardly be alone here. Caladesi Island State Park is home to hundreds of species of birds nesting among the sunflower-flecked small hills of sand. On the other hand, there"s not a parking space in sight.

  How to Get There: The park, off the west coast, can be gotten to by private boats from Honeymoon Island Slate Park.

  Ruby Beach, Washington

  Ruby Beach feels almost mythical (神话的),only when at low tide and in the fog. The rocks there are strange-looking, and the pools can be surprising, too.

  How to Get There: There"s a sign seven miles north of Kalaloch on Hwy. 101. From the parking lot, follow the path through the trees, down a decline, then toward the water.

  Pfeiffer Beach, California

  The rocks there are amazing. In some places, the sand is a light shade of purple. Sure, it can gel windy, but that"s a small price to pay for paradise.

  How to Get There: The turnoff is actually just past the Pfeiffer Big Sur State Park entrance, at Sycamore Canyon Rd. Drive two miles to the beach, and then walk 10 minutes north to Deer Canyon. There"s no sign, but you"ll know it when you see it.

  21. What may happen when the tide comes in?

  A. Long Beach Island is totally gone.

  B. Long Beach Island is popular with tourists.

  C. The footpath to High Bar Harbor can be gone.

  D. The footpath to High Bar Harbor becomes mythical.

  22. What do Ruby Beach and Pfeiffer Beach have in common?

  A. They both look mythical. B. They have special rocks.

  C. The sand there is purple. D. There are some small pools.

  23. Which of the following is TRUE according to the text?

  A. Long Beach Island is the most popular Jersey shores.

  B. You can also enjoy watching many birds and sunflowers at Caladesi Beach.

  C. The rocks at Ruby Beach are obviously fantastic all the time.

  D. You should pay much to enter Pfeiffer Beach, California.

  B

  I first met Joe Gumps when we were both 9 years old, which is probably the only reason he is one of my best friends. If I had first met Joe as a freshman in high school, we wouldn"t even have had the chance lo get to know each other.

  Joe is a day student, but I am a boarding student. We haven"t been in the same classes, sports or extracurricular activities. Nevertheless, I spend nearly every weekend at his house, and we talk on the phone every night. This is not to say that we would not have been compatible if we had first met in our freshman year. Rather, we would not have been likely to spend enough time getting to know each other, due to the lack of immediately visible mutual interests.

  In fact, to be honest, I struggle even now to think of things we have in common, but maybe that"s what makes us enjoy each other"s company so much. When I look at my friendship with Joe, I wonder how many people I’ve known whom I never disliked but sim* didn’t take the time to get to know. Thanks to Joe, I have realized how little basis there is, for the social division that exist in every community.

  Since this realization, I have begun to make an even more determined effort to find friends in unexpected people and places.

  24. Why does the author say Joe Gumps became one of his best friends?

  A. They shared mutual friends in school. B. They had known each other since children.

  C. They shared many extracurricular activities. D. They had many interests in common.

  25. Which of the following can explain the underlined word “compatible” in the passage?

  A. suited and fitted. B. consistent. C. less-connected. D. unsuitable or unfit.

  26. According to the writer, what may be the obstacle for him not to spend time to know others?

  A. That he doesn"t like others" company. B. Being strange to others.

  C. The lack of obvious mutual interests. D. That he’s busy doing other important things.

  27. What has the writer learned from his friendship with Joe?

  A. Durable friendships can be very difficult to maintain

  B. One has to be respectful of other people in order to win respect.

  C. It is hard for people from different backgrounds to become friends.

  D. Social divisions will break down if people get to know each other.

  C

  Olaf Stapledon wrote a book called First and Last Men, in which he looked millions of years ahead. He told of different men and of strange civilizations, broken up by long “dark ages” in between. In his view, what is called the present time is no more than a moment in human history and we are just the First Men. In 2000 million years from now there will be the Eighteenth or Last Men.

  However, most of our ideas about the future are really short-sighted. Perhaps we can see some possibilities for the next fifty years. But the next hundred? The next thousand? The next million? That"s much more difficult.

  When men and women lived by hunting 50,000 years ago, how could they even begin to picture modern life? Yet to men of 50,000 years from now, we may see as primitive(原始的)in our ideas as the Stone-Age hunters do to us. Perhaps they will spend their days gollocking to make new spundels, or struggling with their ballalators through the tribe. These words, which I have just made up, have to stand for things and ideas that sim* can"t think of.

  So why bother even to try imagining life far in the future? Here are two reasons: First, unless we remember how short our own lives are compared with the whole history, we are likely to think our own interests are much more important than they really are. If we make the earth a poor place to live on because we are careless or greedy or quarrelsome, our grandchildren will not bother to think of excuses for us.

  Second, by trying to escape from present interests and imagine life far in the future, we may arrive at quite fresh ideas that we can use ourselves. For example, if we imagine that in the future men may give up farming, we can think of trying it now. So, set your imagination free when think about future.

  28. A particular mention made of Stapledon"s book in the opening paragraph .

  A. shows the popularity of the book B. serves as a description of human history

  C. serves as an introduction to the discussion D. shows a disagreement of views

  29. The text discusses men and women 50,000 years ago and 50,000 years from now in order to show that .

  A. human history is extremely long B. it is useless to plan for the next 50 years

  C. life has changed a great deal D. it’s difficult to tell what’ll happen in the future

  30. "Spundels" and "ballalators" are used in the text to refer to .

  A. unknown things in the future B. ideas about modem life

  C. tools used in farming D. hunting skills in the Stone-Age

  31. According to the writer of the text, imagining the future will .

  A. help us to improve farming B. enable us to better understand human history

  C. serve the interests of the present and future generations D. make life worth living

  D

  Nowadays, parents are faced with more pressure while bringing up their kids, we want our children to reach their full academic potential. We read to them, encourage their special talents, and support them when they have problems. If they choose to participate in music or sports, we also help them reach their potential in those areas. These are all good goals.

  There is, however, an even more important goal. It is a goal more difficult than excellence in arithmetic or soccer or the violin. Parents are responsible for providing their children with a moral compass. They need to nurture and treasure goodness in their children. Every child has the capacity to become a good, decent human being. To fulfill this capacity, children need the guidance and support of parents and other adults. Raising good, moral children is the most important job we will have.

  What is a good, decent human being? While we may differ on some details, most would agree that respect for others, kindness and caring, honesty and honor, and a reverence (尊重)for life are key. Good, decent human beings are people with a firm sense of direction and purpose, a moral compass to guide their lives. Children need our help to develop these characteristics and values.

  However, goodness is not easy for an adult, and it is even more difficult for a child. They do not have the knowledge, experiences, or cognitive skills to understand the impact and consequences of their actions.

  Reinforcement is sometimes an efficient approach to building positive behavior. The child behaves in a desired way and the parents provide reinforcement. The behavior then typically increases in frequency. This approach works for teaching "Please" and “Thank you". The new behavior is maintained, because it helps the child get along in the world.

  Loving, everyday interactions are the beginning of raising moral children. The child falls and receives adult concern learns how to treat others kindly when they stumble(绊倒).The child who makes a mistake and is encouraged to try again learns how to support others. When parents intercede (调解)graciously for their child, the child can see the basis for friendship. When we treat children with respect and care, we provide a positive model of how to behave. These early experiences establish patterns for their treatment of others.

  32. What are adults supposed to do to make children become good, decent human beings?

  A. support their children when they have problems

  B. encourage their children to be a man with high honesty

  C. set a good example and give positive instructions to children

  D. encourage their children to learn more knowledge about sports or music

  33. Why is it difficult for a child to act with honor and kindness?

  A. Because it is difficult for adults to do so.

  B. Because a child is not mature enough to do so.

  C. Because a child does not understand the abstract concept of “goodness".

  D. Because a child does not have a model to follow.

  34. Why does the author talk about leaching "Please" and “Thank you" in Paragraph 5?

  A. To demonstrate that reinforcement is an efficient approach to building positive behavior

  B. To demonstrate that politeness is the nature of a moral compass.

  C. To demonstrate that politeness helps the child gel along in the world.

  D. To demonstrate that saying "Please" and “Thank you" is a good habit.

  35. Which of the following can express the main idea of the last passage?

  A. Adults should treat children with love.

  B. Adults should have more positive communication with children.

  C. Adults should treat children with respect and care.

  D. Adults should provide a positive model of how to behave for children.

  第二节(共5小题,每小题2分,共10分)

  根据短文内容,从短文后的选项中选出能填入空白处的最作选项。选项中有两项为多余选项。

  Do you have trouble finding something to do that your children can enjoy? Don"t worry! These activities make family outings perfect for weekends.

  36

  The next time your child"s favorite band is in town, take him to a show for a special treat. Tickets may be hard to obtain, especially for a famous concert. If you"re really unable to get tickets, check the local theater for them.

  Science museum

  If you live in a large city, chances are that you have a science museum nearby. 37 Be sure to catch a lecture, live performance, or a show for fascinating subjects. Be sure to be close to some firsthand and fantastic science experiments.

  The zoo

  Take the whole family on a trip to the local zoo. 38 Little kids love to stop by the zoo to enjoy a variety of friendly animals. Older kids will enjoy sitting here to learn about the various environments and eating habits of wild animals.

  Live sporting event

  Who doesn"t love a live sporting event? Take me out to the ball game! Support your favorite professional team with a family outing to the stadium. 39 And root your team on to a win.

  Camping

  Become one with nature. Ask your children to identify the different animal and insect noises, or collect backyard materials for a special project. 40 It"s amazing how your own yard can be transformed into a nature wonderland(奇境)at night!

  A. Kids" concert or show

  B. The cinema

  C. Satisfy your child"s curiosity

  D. When do you watch a football game?

  E. Grab a hot dog and some peanuts, then settle onto your seat.

  F. Visit each creature’s exhibit and test your kids’ animal knowledge.

  G. At bedtime, rest in sleeping bags under the stars, or set up a tent.

  第三部分:英语知识运用(共两节,满分45分)

  第一节:完型填空(共20小题:每小题1.5分,满分30分)

  阅读下面短文,从短文后各题所给的A、B、C和D四个选项中,选出可以填入空白处的最佳选项,并在答卡上将该项涂黑。

  One day, during the long summer holiday, Mrs Martins took Bredon and his younger brother to the beach. As soon as they got there, the two children 41 down to the sea to try out their new surfboats. They were soon swimming out to the 42 and riding back on their boards.

  After a while, Brendon noticed that Kim was 43 away from him.

  “Come back, Kim,” he shouted. “Mum said we have to stick together.” Kim put his arms over the side of his board and began paddling(划水), but instead of getting 44 he began moving further away.

  “Brendon, help!” called the young boy. “I’m 45 in a trip current(退潮流).” Brendon quickly paddled over to his 46 .

  “Hold 47 to the back of my board and I’ll pull you in,” he told the 48 boy. But the trip was

  too 49 and soon both Brendon and Kim were moving still further from the beach.

  “I cann’t even see our beach any more,” 50 Kim.

  “Yes, 51 we’re not moving out to sea any more. We’re just floating along the coast. See, there’s the next beach,” said Brendon 52 , “We’re moving closer to it.” Sure enough, the current was now taking them closer to the beach and becoming 53 .

  “Right, now’s the time to paddle,” said Brendon, suddenly full of 54 . Before long the children had paddled to the beach 55 they sat for a few moments to 56 their breach. Then they 57 up their boards and started the long walk back to their mother. When they got there, they were surprised to find the beach full of people and boats.

  “Thank goodness!” cried their mother running up to 58 them.” ‘We were just about to start a search. One minute you were in the waves in front of me and the next you were gone. I was so 59 .”

  “It’s OK,” said Brendon, bugging his mother.” “We did just as you told us and 60 together.”

  41.A. raced B. got C. swam D. jumped

  42.A. beaches B. fields C. waves D. sands

  43.A. turning B. breaking C. floating D. pulling

  44.A. shorter B. higher C. faster D. closer

  45.A. found B. trapped C. absorbed D. rested

  46.A. mother B. brother C. father D. sister

  47.A. on B. in C. up D. off

  48.A. determined B. frightened C. surprised D. interested

  49.A. hard B. mild C. strong D. soft

  50.A. announced B. claimed C. smiled D. cried

  51.A. or B. so C. and D. but

  52.A. carelessly B. sadly C. cheerfully D. attentively

  53.A. weaker B. clearer C. lower D. faster

  54.A. spirit B. energy C. source D. force

  55.A. how B. where C. why D. when

  56.A. make B. feel C. hold D. catch

  57.A. picked B. mixed C. rolled D. fixed

  58.A. punish B. beat C. hug D. scold

  59.A. depressed B. amazed C. embarrassed D. worried

  60.A. stuck B. sank C. shock D. struck

  第Ⅱ卷(非选择题 共50分)

  第二节 语法填空(共10小题,每小题1.5分,满分15分)

  阅读下面材料,在答题卡上填入适当的内容(1个单词)或括号内单词的正确形式。

  “Do you call that a hat?” I said to my wife.

  “You needn’t be so rude about it,” my wife answered as she looked at 61. (she) in the mirror.

  I sat down on one of those modern chairs with holes in it and 62. (wait). We 63. (be) in the hat shop for half an hour 64. my wife was still in front of the mirror.

  “ We mustn’t buy things we don’t need,” I remarked 65. ( sudden). I regretted 66. (say) it almost at once.

  “ You needn’t have said that, ”my wife answered. “I needn’t remind you 67. that terrible tie you bought yesterday.”

  “ I find 68. beautiful,” I said. “A man can never have too many ties.”

  “ And a woman can’t have too many hats, she answered.

  Ten minutes 69. we walked out of the shop together. My wife was wearing a hat 70. looked like a lighthouse!

  第四部分 写作(共两节,满分35分)

  第一节 短文改错(共10小题,每小题1分,满分10分)

  假定英语课上老师要求同桌之间交换修改作文,请你修改你同桌写的以下作文。文*有10处语言错误,每句中最多有两处。每处错误仅涉及一个单词的增加、删除或修改。

  增加:在缺词处加一个漏字符号(Λ),并在其下面写出该加的词。

  删除:把多余的词用斜线(\)划掉。

  修改:在错的词下画一横线,并在该词下面写出修改后的词。

  注意:1.每处错误及其修改均仅限一词;

  2.只允许修改10处,多者(从第11处起)不计分。

  Dear Li Ming,

  The summer holiday is around corner. Do you have any plan for them? I plan to read some English book during the holiday and I needed your recommendations.

  Firstly, I want to improve my English level, hope one day I can go to abroad for further study in other countries, such as America and England. Secondly, I also desires to know more history, geography as good as cultures and traditions about these countries. But can you make a list of the books of me to read? Thank you sincerely in advance.

  I am looking forward to your re*.

  Li Hua

  第二节 书面表达(满分25分)

  假定你是李华,贵校在学生中征集意见,询问学生是否赞成开设iPad课程(iPad classrooms)。你支持开设iPad课堂;请给校长写一封建议,要点如下:1.资源丰富;2.促进交流;3.利于环保;4.其他理由.

  注意:1.词数120左右;2.可以适当增加细节,以使行文连贯;3.开头语已为你写好。

  Dear Headmaster,

  We have been asked about our opinions on opening iPad classrooms.

  Yours sincerely,

  Li Hua


英语高考北京模拟试卷及答案(扩展7)

——广东市高考语文模拟试卷及答案

广东市高考语文模拟试卷及答案1

  一、现代文阅读(35分)

  (一)论述类文本阅读(9分)

  阅读下面的文字,完成1—3题。

  文字是记录和传达语言的书写符号。语言是有声的思维,文字是有形的思维。二者都是思维的外化。像语言一样,文字也是思维能力的反映,蕴含着思维的特点。从某种意义上说,古文字是对思维进行考古的工具。

  古人类主要运用形象进行思维。这种特点反映在古诗歌中,人们喜欢用比兴来说明某种道理;反映在原始神话中,人们通过具体的故事来说明某个道理,抽象的道理是没用的。形象思维曾是人类最得心应手的思维方式。抽象思维是在形象思维的基础上逐步发展起来的。从文字的创造特点看,绘画文字是最早的文字。这种文字的主要特点是象形。不仅中国的古文字用形象来表示,古巴比伦人、古雅玛人、古埃及人等无一不是从象形开始来创造古文字的。

  创造文字的时代,人们抽象思维的能力还比较薄弱。人们进行抽象思维,还很难脱离具体形象来进行,往往要依赖于对形象的类比、比较、联想来进行。表现在文字的创造上,形与义密切结合在一起,人们是从形象来了解符号的意义的。文字的产生一方面是以人类抽象思维的能力达到一定水*为基础的。但另一方面,象形文字又标志着人类抽象思维能力还不发达,这种低下的抽象思维能力不可能创造太抽象的符号。

  抽象思维能力的低下还主要表现在古文字上的词类上,表现为形容词的极端贫乏。从甲古文看,形容词数量相当少,只有白、厚、明、光、赤、美、丽等为数不多的形容词。事物的性质包含在事物之中,与事物不可分离,只能通过人的思维才能把事物的性质从事物本身分开。在近代一些少数民族那里,要说热,只能说“像火一样”。

  早期文字中的“鱼”字是各种各样的鱼类象形,后来逐渐概括出一种共同的鱼的特征的象形文字,作为各种各样鱼的代表。早期文字中,几乎每个文字都经历了一定的概括过程。没有概括过程,就不可能产生一般意义上的文字。这种概括能力使人们能概括出事物的共同性质,撇开了同类事物之间的差别性、个性和偶然性,产生了最初的种、类概念。

  在概括能力发展的同时,形象思维本身也在不断发展着。形象思维从自身中演化出了抽象思维。抽象思维的发展又推动着形象思维的发展。古人类从事物中抽象出事物的性质,从而创造了形容词后,又创造着代表新的形象的词。例如,从羊的肥大的形象创造了“美”字,进一步又创造了“美服”“美女”等新的名词。形容词产生后,也使原有的一些代表具体形象的词成为更形象生动的新词。例如月字、镜字,在抽象出了“明”字后变成了“明镜”“明月”,使形象更逼真。古人类从众多的个别的事物中概括出代表一般意义的词后,又以此为指导去认识新的个别事物,创造新的字。

  从古文字的演变进化过程中,我们可以看到形象思维与抽象思维在互相补充、互相促进中辩证地发展,使人类的思维能力不断由低级向高级发展,从而使人类能够更探更广地认识世界。

  (摘编自朱长超《从古文字看原始思维及其发展》,有删改)

  1.下列关于原文内容的表述,不正确的一项是(3分)

  A.“语言”与“文字”都是思维的外化,如同可以通过古代语言来考察原始思维一样,我们也可以从古文字来考察原始思维。

  B.创造文字的时代里,人们常需要依赖形象的类比、比较、联想来进行抽象思维。那时候,人们从形象来了解符号的意义。

  C.概括让人们认识到了事物的共性,没有概括就不可能产生一般意义上的文字,因而几乎每个文字都经历了一定的概括过程。

  D.古人类充满智慧,能够从众多个别的事物中概括出代表一般意义的词之后,又以此为指导去认识新的个别事物,创造新字。

  2.下列理解和分析,不符合原文意思的一项是(3分)

  A.因为文字是有形的思维,是思维能力的反映,蕴含着思维的特点,所以从某个意义上而言,古文字是对思维进行考古的工具。

  B.人们在古诗歌中喜欢用比兴来说明某种道理,在原始神话中用具体故事来说明某种道理。这种形象思维的主要特点便是象形。

  C.事物的性质与事物不可分离。形容词是对事物性质的抽象概括,当人们具备一定的抽象思维能力之后,形容词才有可能出现。

  D.形象思维与抽象思维关系密切、辩证发展。形象思维从自身中演化出了抽象思维,抽象思维的发展又推动着形象思维的发展。

  3.根据原文内容,下列理解和分析不正确的一项是(3分)

  A.古文字是考察原始思维的宝贵资源,因而,当下对古文字的研究与保护,不仅是对人类文化的尊重,更是对人类历史的尊重。

  B.古中国人、古巴比伦人、古雅玛人、古埃及人都是从象形开始创造古文字。这说明古代东西方人的思维中存有相同的因子。

  C.文字的产生是以人类抽象思维能力达到一定水*为基础的,因而,如果人类抽象思维的能力不太发达,人类就不太可能创造出文字。

  D.抽象思维与形象思维在相互补充、相互促进中辩证地发展,推动人类思维能力不断发展,使人类更好地认识世界。

  (二)实用类文本阅读(12分)

  阅读下面的文字,完成4—6题。

  梁思礼与梁启超

  解璋玺

  梁思礼生于1924年8月24日。这一年,或可称为梁启超的“灾年”。他的第一任夫人李蕙仙因病离世,而丧事初了,子又远行留学。因而老年得子,算是给他一点难得的慰藉,于伤痛之中还能体会到人生未尽之乐。

  梁启超固深爱着他的九个儿女,但爱小儿子梁思礼尤甚。这种人之常情是不难为人所理解的。他称小儿子“老baby(老白鼻),在写给儿女的信中常常提到这个聪明可爱、讨人喜欢的小儿子,以至于姐姐、哥哥们责怪“爹爹信中只说老白鼻不说别的弟妹,太偏心”,结果,梁启超特意在信中写了一大段文字,专讲达达、六六、司马懿(七女)的近况。不过,梁启超并不回避他是“专一喜欢老白鼻”的。他注意到老白鼻一点一滴的进步和变化,不到两岁,就已经“一天到黑‘手不释卷’”了,而且能唱“葡萄美酒”。梁启超在饮冰室读书、写作时,是不允许孩子们来打扰的,但老白鼻是个例外,每天“总来搅局几次”,梁启超不仅不恼,甚至“盼望老白鼻快来”,因为老白鼻的到来,恰恰是他“最好的休息机会”。过生日了,大孩子都不在身边,他就叫老白鼻代表姊姊、哥哥们拜寿。老白鼻也乐得如此,居然“一连磕了几十个响头,声明这是替亲家的,替二哥、三哥乃至六姊的”,而作为父亲,他在信中告诉大孩子们,“我都生受你们了”。这种父子之间其乐融融的场面,不仅儿子是天真烂漫的,父亲又何尝不是和蔼可亲,一团孩子气呢。

  梁启超病逝时梁思礼只有四岁半。母亲与姊姊、哥哥们合力供他读完了小学、中学,太*洋战争爆发前夕,母亲筹措了四百美金,送他到美国求学。其实,梁启超早在1925年为几个孩子预筹学费时曾说过:“至于老白鼻那份,我打算不管了。到他出洋留学的时候,有姊姊哥哥,还怕供给他不起吗?”然而,中日战争爆发后,他的姊姊、哥哥们陆续流落大后方,自顾不暇,哪里还有余力顾及远在天津的这个小弟弟?

  梁思礼到美国后,考入普渡大学电机工程系。由于家道中落,要靠家庭资助完成学业已不可能,他只能靠自己打工赚钱,交学费和维持生活。 “他凭着自己的聪明才智和苦斗精神,在国外打工读书,在饭馆里洗碗碟,在游泳池当救生员,他什么都能干”,苦读了八年。在1945年从普渡大学毕业,获得学士学位,随后,他又获得辛辛那提大学硕士学位和博士学位。梁家子弟没有不是心怀报国热忱的,虽然著名的无线电公司RAC向他发出了邀请,并许诺优厚的待遇,但父亲的话言犹在耳:“人必真有爱国心,然后方可用大事。”于是,他毅然谢绝了美国公司的邀请,和姊姊梁思懿同船回到祖国,投身于新中国的建设。

  几十年来,呕心沥血,筚路蓝缕,屡败屡战,排除万难,终于将远程运载火箭研制成功。中国能够成为世界上的航天大国,梁思礼作为开拓者中的一人,功不可没。

  1987年,他被选为国际宇航科学院院士,并担任国际宇航联合会副*;1993年,又当选为中国科学院院士,成为梁氏一门三院士①中的一位。

  (选自《新华文摘》,有删改)

  【注】①一门三院士,九子皆才俊。梁启超有九个子女,除梁思礼外,还有两位院士:梁思成,建筑学家;梁思永,考古学家,均为中央研究院院士。此外,梁思顺,诗词研究专家;梁思忠,西点军校毕业,参与淞沪抗战;梁思庄,图书馆学家;梁思达,经济学家;梁思懿,社会活动家;梁思宁,曾参加新四军。

  4.下列对材料有关内容的概括和分析,最恰当的一项是(3分)

  A.1924年是梁启超的“灾年”:妻子因病去世,丧事初了,爱子远行留学。梁思礼的出生,消解了他所有的抑郁情绪。

  B.梁思礼也许没能直接受教于梁启超,但梁启超给他的童年创造了一个健康、快乐的成长环境。他从父亲的言传身教中自会受到春风化般的滋润和熏陶。

  C.梁启超在写给儿女的信中常常提到讨人喜欢的小儿子,以至于哥哥、姊姊们责怪父亲不关心出国的子女,但是,梁启超没有作回应。 [来源:学,科,网Z,X,X,K]

  D.梁思礼历时几十年将远程运载火箭研制成功,使中国成为了世界上的航天大国。同时,因贡献突出,被评为中国科学院院士,成为梁氏一门三院士中的一位。

  5.结合全文,请简要分析梁思礼具有哪些精神品质?(4分)

  6.每当我们追念起梁启超继而谈到“一门三院士,九子皆才俊”时,心中总是为梁启超对儿女的成功教育而叹服。请结合文本谈谈你的看法。(5分)

  (三)文学类文本阅读(14分)

  阅读下面的文字,完成7—9题。

  借给你的温柔

  江文胜

  这天晚上,天下大雪,出奇的冷。养猪专业户王老汉蜷缩在家里的火盆前,他不敢出门。

  忽然,狗叫。王老汉极不情愿地打开屋门,一股刺骨的寒风迎面刮来,他打了一个激灵。王老汉哆哆嗦嗦地来到院子里,没见到任何人,可狗还在叫。

  院子左边的一排房屋,是猪圈。他逐一查看猪圈,仔细地数,总共60头猪连毛发都没少一根。猪没被盗,狗为啥叫呢?肯定有情况,不然,通人性的狗是不会无缘无故地瞎叫的。果然,王老汉在那排距自己的住房最远的饲料库发现了“敌情”:喂猪的苕片饲料少了一袋!

  王老汉立马来了精神,他赶紧回到屋里,拉上三十岁出头的儿子金华出来寻找。原来下了雪,现在雪停了,院子里从院门到饲料库、从饲料库到院门各有两行脚印,清晰可辨。金华说:“咱们跟着脚印去追那偷饲料的人!”王老汉说:“这是一个人的脚印,咱俩去追,可以对付,铁证如山,看他如何狡辩。”

  父子俩一前一后顺着脚印一口气追到了蜡子山矿区宿舍。脚印通到一间砖房的门前没了,别的砖房都熄了灯,只有这间房没有熄灯。

  真是胆大包天啊!做了贼还敢亮着灯?王老汉这样想。

  窗是玻璃窗,玻璃上结满了冰花。左下角的一块玻璃没了,糊上了报纸。王老汉用舌头舔湿了窗户纸,用手指轻轻一捅,报纸上便出现了一个小孔:一个黑黑的瘦瘦的中年汉子,坐在屋里的矮凳上,他的头上还冒着热气。啊!那袋装有红苕片的大编织袋赫然放在他的面前。床沿坐着一个双手笼在袖头里的老头,床头的被子下露出一个小脑袋,一个中年妇女站在这个黑汉子的对面。

  王老汉父子猫着腰蹲在窗户外,大气不敢出,倾听着屋里的动静。

  “粮,借来了。这就是,红苕片,100斤。你还愣着干啥?还不快去熬苕片汤!”汉子的声音。

  “妈,快熬,我饿坏了,我要吃苕片汤。”小孩的声音。

  “亚龙,这粮是咋借来的?”老人的声音。

  “爹,我事先写好了借条,借条压在袋子下。”汉子的声音。

  王老汉一愣,心想,我怎么没注意到那借条!真的有?

  “这不是偷吗?自打你从娘胎里出来,我是怎么指教你的?做人要本分!还好,留下了借条,人要凭良心。但借粮容易还粮难哪!矿上不开工资,啥时候能还上?”老人的声音。

  “爹,这个我想过。车到山前必有路,活人不能让尿憋死,我还有一双手。到春天,矿上如果还不能开支,我就到农村去帮工。听说王老汉厚道,日子也过得殷实,不然*吗要借他的。我想去他那里,帮他干活,用工钱抵粮钱……”汉子的声音。

  “快走!”王老汉在心里喊。他拉起儿子往家里赶。进了院子,王老汉急奔饲料库,果然发现了那张借条:

  家里无米下锅。借一袋红苕干片。春天还。蜡子山矿区吴亚龙,此据。

  某某年某月某日

  上面还有一个用墨水作印泥盖的“吴亚龙”的私章。

  太阳从蜡子山上露出头来。吴亚龙推开屋门,见门口堆了一些东西:一袋大米、一袋面粉、一大块猪肉、几条鲜鱼和一袋冻豆腐……吴亚龙像见到了天外来物似的翻来覆去地找,竟找不出一张哪怕是手指宽的字条……

  7.下列对这篇小说思想内容和艺术特色的分析,最准确的一项是(3分)

  A.小说以“借给你的温暖”为题,揭示了文章的主题,表现了淳朴的人性美。

  B.小说运用了外貌、神态、动作、心理、对话、细节等描写手法,生动形象地刻画了王老汉和吴亚龙的形象。

  C.这篇小说讲了一个感恩的故事,他把人物放在大饥饿的时代,用简洁的文字给人物设置了一个道德困境。

  D.小说的结尾照应了题目,既出人意料,又在情理之中;而且,使得王老汉“诚信”的形象更为突出。

  8.小说中两处写到“雪”,分别有何作用?请结合文本简析。(5分)

  9.小说中王老汉和吴亚龙,哪一位是主人公?请结合文本谈谈你的看法。(6分)

  二、古代诗文阅读(35分)

  (一)文言文阅读(19分)

  阅读下面的文言文,完成10—13题。

  周起字万卿,淄州邹*人。生而丰下,父意异之,曰:“此儿必起吾门。”因名起。幼敏慧如*。意知卫州,坐事削官,起才十三,诣京师讼父冤,父乃得复故官。举进士,授将作监丞。擢著作佐郎,累迁户部度支判官。真宗北征,领随军粮草事,寻为东京留守判官,判登闻鼓院。以右正言知制诰,权判吏部流内铨。封泰山,摄御史中丞,所过得采访官吏能否及民利病以闻。东封还,近臣率颂功德,起独以居安为戒。初置纠察刑狱司,因命起,起乃请诸已决而事有所枉及官吏非理榜掠者,并听受诉,从之。擢枢密直学士、权知开封府。起听断明审,举无留事。真宗尝临幸问劳,起请曰:“陛下昔龙潜于此,请避正寝,居西庑。”诏从之,名其堂曰继照。起尝奏事殿中,适仁宗始生,帝曰:“卿知朕喜乎?宜贺我有子矣。”即入禁中,怀金钱出,探以赐起。从祀汾阴,权知河中府,徙永兴、天雄军,所至有风烈,数赐书褒谕。拜给事中,进礼部侍郎。起素善寇准,尝与寇准过同列曹玮家饮酒,既而客多引去者,独起与寇准尽醉,夜漏上乃归。明日入见,引咎伏谢,真宗笑曰:“天下无事,大臣相与饮酒,何过之有?”准且贬,起亦罢为户部郎中,又降太常少卿。后复为礼部侍郎,以疾请知颖州,徙陈州。卒,赠礼部尚书,谥安惠。起性周密,凡奏事及答禁中语,随辄焚草,故其言,外人无知者。家藏书至万余卷。起能书。弟超,亦能书,集古今人书并所更体法,为《书苑》十卷。

  (节选自《宋史•列传第四十七》)

  10.对下列句子中加点字的解释,不正确的一项是(3分)

  A.生而丰下,父意异之 认为……奇特

  B.摄御史中丞 代理

  C.擢枢密直学士、权知开封府 暂且

  D.起素善寇准 好好地

  11.下列对文中加点词语的相关内容的解说,不正确的`一项是(3分)

  A.庙号是古代帝王、大臣等死后,据其生*事迹评定的称号,如真宗、仁宗。

  B.泰山封禅起源于春秋战国时期,是帝王受命于天的典礼,目的是巩固皇权,粉饰太*。

  C.礼部为六部之一,掌管礼仪、祭祀、学校、科举等,长官为礼部尚书。

  D.我国古代以铜壶滴漏计算时间,夜漏就是深夜时分。

  12.下列对原文有关内容的概括和分析,不正确的一项是(3分)

  A.周起关注民生,思虑长远。他把采访官吏能力大小及百姓疾苦报知朝廷;东封还朝后,皇上近臣都歌功颂德,唯有周起认为应该居安思危。

  B.周起善于断案,明辨是非。担任纠察刑狱司官员时,周起倾听了那些已经判决但事实确有冤情,以及官吏胡乱拷打的犯人的申诉,受到皇上慰劳。

  C.周起交好寇准,受到牵连。他一向与寇准关系好,曾到寇准家喝酒,尽醉方归;寇准将要被贬官,周起也被连续降职。

  D.周起为人谨慎,做事周密。凡是上奏事情以及书面应答宫中的文书,周起随后就烧掉草稿,他的言语看法,外人没有知晓的。

  13.把文中画横线的句子翻译成现代汉语。(10分)

  (1)意知卫州,坐事削官,起才十三,诣京师讼父冤,父乃得复故官。

  (2)从祀汾阴,权知河中府,徙永兴、天雄军,所至有风烈,数赐书褒谕。

  (二)古代诗歌阅读(11分)

  阅读下面这首宋词,完成14-15题。

  西江月①

  【宋】苏轼

  照野弥弥②浅浪,横空隐隐层霄③。障泥④未解玉骢骄,我欲醉眠芳草。

  可惜一溪风月,莫教踏碎琼瑶⑤。解鞍欹枕绿扬桥,杜宇一声春晓。

  【注】①此词为苏轼被贬黄州作。其春夜行蕲水边,过酒家,饮醉,乘月至一溪桥上,解鞍,枕臂醉卧少休。拂晓,见乱山攒拥,流水淙淙,宛如仙境,书此词于桥柱上。②弥弥:水波翻动的样子。③层霄:弥漫的云气。④障泥:马鞯,垂于马两侧以挡泥土。⑤琼瑶:美玉。这里形容月亮在水中的倒影。

  14.下列对本词的理解,不正确的两项是(5分)

  A.“障泥未解玉骢骄,我欲醉眠芳草”,写词人频临溪流,从马上下来,等不及卸下马鞯,即欲眠于芳草。既侧面描绘出月下溪景之美,又表现了词人的喜悦心情。

  B.“可惜一溪风月,莫教踏碎琼瑶”一句运用了借代的修辞手法。琼瑶,本是美玉,这里代指月色,水月交辉,有如晶莹剔透的珠玉。此句传神地写出水月之静美。

  C.“解鞍欹枕绿杨桥,杜宇一声春晓”一句用“解鞍欹枕”这个特写镜头表现出词人的随意豁达,而“杜鹃春晓”写出空山春晨的喧闹和生机。

  D.本词写作者爱惜一溪风月,不让马踏碎溪月,解鞍酣眠直至春晓,描写了一个醉心自然、随性洒脱、乐观豁达的诗人形象。

  15.请从表达手法的角度简要赏析“照野弥弥浅浪,横空隐隐层霄”两句的妙处。(6分)

  (三)名篇名句默写(5分)

  16.补写出下列句子中的空缺部分。(5分)

  (1)在《念奴娇•赤壁怀古》中词人神游故国后,用“ , ”来书写自己多愁善感而顿觉年华逝去的感叹。

  (2)在《生于忧患死于安乐》中,孟子认为上天要让一个人担负重大使命,一定会让这个人经历种种艰难困苦,最终“ , 。”

  (3)在《永遇乐•京口北固亭怀古》中,辛弃疾回顾了元嘉年间的那次北伐,宋文帝刘义隆本希望能够“封狼居胥”,但是由于行事草率,最终却“ ”。

  第Ⅱ卷 表达题(80分)

  三、语言文字运用(20分)

  17.下列各句中加点成语的使用,全都正确的一项是(3分)

  ①从历史上看,人类社会每一个新时代的开启,似乎都与技术变革息息相关。如今,正是新的技术革命爆发的前夜。

  ②他心高气傲,目空一切,总喜欢妄自菲薄别人,结果可想而知,没有人愿意跟他打交道,他成了大海里的一叶孤舟。

  ③在义工联盟中,他不仅是发起人、带头者,更是义工精神的实践者、坚守者,时时处处都处心积虑,亲力亲为。

  ④82岁高龄的袁隆*面对辉煌没有任何改变,他依然风尘仆仆地骑着摩托车去实验田,依然从北到南查看育种基地。

  ⑤莫言小说最先征服你的并不是故事和人物,而是语言。那一个个*淡的文字背后深藏着穿云裂石的哀痛和精彩斑斓的怜爱。

  ⑥自2015年6月起,经过三个月的改造,仁和花园焕然一新。如今,小区活动广场内篮球场、健身器材、桌椅等一应俱全。

  A.①④⑥  B.②③⑥ C.①④⑤   D.③⑤⑥

  18.下列各句中,没有语病的一句是(3分)

  A.峰会期间新闻中心共接待大约来自近70个国家中外媒体记者共1.8万人次,并成功举办12场新闻发布会。

  B.在里约残奥会上,中国残奥代表团共获107枚金牌、81枚银牌和51枚铜牌,239块奖牌的骄人成绩连续第四次占据金牌榜与奖牌榜双第一。

  C.根据国家统计局9月9日发布的数据显示,8月份CPI(全国居民消费价格总水*)同比上涨1.3%,创下今年以来新低。

  D.日前,北京市公布了《北京市关于进一步推进户籍制度改革的实施意见》。至此,全国已出台户籍制度改革方案的省份达到30个。

  19.填入下面文段空白处的词语,最恰当的一组是 (3分)

  爱因斯坦的左脑特别发达,__①__他总是使用左脑。他分析自然现象的能力和形成观念的能力非常卓越。__②__有趣的是,他__③__用语言__④__用非语言(比如图形)进行思考的,__⑤__是以一种跳跃的方式。完成之后,再将它转换成语言,__⑥__从左脑转到右脑。这就是“思维错位症”。

  ① ② ③ ④ ⑤ ⑥

  A 因为 但 不是 而是 而且 即

  B /[ 并且 不仅 而且 就是 即

  C 但是 但 不是 而是 并且 然后

  D 因为 并且 不仅 而且 就是 然后

  20.在下面一段文字横线处补写恰当的语句,使整段文字语意完整连贯,内容贴切,逻辑严密。每处不超过15个字。(5分)

  “字”是我们工作及生活中必不可少的部分,那么,① ?关于汉字的起源,一直是我国学者研究的对象,但一直② ,直到最近几十年,我国考古学家才先后发布了一系列“安阳殷墟甲骨文”及与汉字起源有关的出土资料。资料显示,我国最早的汉字可追溯到原始社会晚期及人类社会早期在陶器上面刻画或彩绘出的符号,同时,还有少量刻写在甲骨、玉器、石器等上面的符号。这些符号,距今已经有8000多年历史。由此可见,③ 。

  21.下面是某宾馆住客控制流程图,请把这个构思写成一段话,要求内容完整,表达准确,语言连贯,不超过60字。(5分)

  四、写作(60分)

  22.阅读下面的材料,根据要求写一篇不少于800字的文章。(60分)

  林丹和李宗伟,一个是金牌拿到手软,光世界冠军就有将近20个的大满贯得主;一个是穷其一生可能也无法实现奥运冠军梦的“千年老二”。我们会记得林丹夺冠后的疯狂,也有不少人忘不了李宗伟落寞的身影。有网友说,若没有李宗伟,林丹纵然再强,也无法被激发出最好的自己。林丹说:“你们除了看到我以外,也应该看到我的对手李宗伟,我不觉得他是失败的。他通过努力让很多人也认可他,我觉得这就足够了。金牌只是一种标准,不能够代表所有。有时我也会向李宗伟学习,我会想他输给了我这么多次重要比赛,为什么还能够放下一切,继续跟我再拼下一次比赛?”李宗伟在接受采访时说:“其实冠军对我来说已经不是很重要了,我想的是接下来看能不能再多打两年,慢慢享受羽毛球吧。”

  要求:选好角度,确定立意,明确文体,自拟标题;不要脱离材料内容及含意的范围作文,不要套作,不得抄袭。

推荐访问:高考 英语 北京 英语高考北京模拟试卷及答案 英语高考北京模拟试卷及答案1 北京高考英语模拟题 高考英语试卷北京卷 2017北京高考英语试卷及解析

本文标题:英语高考北京模拟试卷及答案(范例推荐)
链接地址:https://www.senjie2201.com/zhuantifanwen/gongwenfanwen/82299.html

版权声明:
1.世霆文库网的资料来自互联网以及用户的投稿,用于非商业性学习目的免费阅览。
2.《英语高考北京模拟试卷及答案(范例推荐)》一文的著作权归原作者所有,仅供学习参考,转载或引用时请保留版权信息。
3.如果本网所转载内容不慎侵犯了您的权益,请联系我们,我们将会及时删除。

关于世霆文库网 | 在线投稿 | 网站声明 | 联系我们 | 网站帮助 | 投诉与建议 | 人才招聘 |

Copyright © 2017-2024 世霆文库网 Inc. All Rights Reserved. 版权所有

本站部分资源和信息来源于互联网,如有侵犯您的权益,请尽快联系我们进行处理,谢谢!备案号:沪ICP备17000782号-1